SUSIEG 2018 A Flashcards

1
Q

An elderly patient with hypothyroidism is recovering from a fall and is referred to physical therapy to increase exercise tolerance and safety. The patient complains to the therapist of significant muscle pain in both lower extremities. What additional musculoskeletal effects should the therapist examine for?
A. Distal muscle weakness
B. Proximal muscle weakness
C. Joint laxity
D. Decreased deep tendon reflexes

A

Correct Answer: B
Hypothyroidism can have numerous musculoskeletal effects, including myalgia (muscle pain) and proximal muscle weakness.

How well did you know this?
1
Not at all
2
3
4
5
Perfectly
2
Q

A patient is referred to a woman’s health clinic with moderate to severe uterine prolapse.
What symptoms should the therapist examine for?
A. Absent perneal sensation
B. Bowel leakage
C. Low back pain and perineal discomfort aggravated by prolonged standing
D. Low back pain and perineal discomfort aggravated by lying down

A

Correct Answer: C
Low back pain and perineal discomfort aggravated by prolonged standing are common with uterine prolapse.

How well did you know this?
1
Not at all
2
3
4
5
Perfectly
3
Q

A child experienced a superficial partial-thickness burn from a scalding pot of water affecting 26% of the thorax and neck. On what should the therapist’s INITIAL plan of care focus?
A. Return to preborn function and activities of daily living
B. Pain management
C. Infection management
D. Chest wall mobility and prevention of scar contracture

A

Correct Answer: D
Prevention of scar contracture and preservation of chest wall mobility and normal neck range of motion (ROM) are the initial major goals to focus on with this patient.

How well did you know this?
1
Not at all
2
3
4
5
Perfectly
4
Q

A patient with coronary artery disease has been doing regular aerobic exercise on a treadmill. If the patient fails to comply in taking prescribed beta-blocker medication and continues to exercise, what potential rebound effect could result?
A. Increase in blood pressure and decrease in heart rate during exercise
B. Decrease in blood pressure and heart rate during exercise
C. Increase in blood pressure and heart rate during exercise
D. Decrease in blood pressure and increase in heart rate during exercise

A

Correct Answer: C
Beta-blockers affect the beta-1 adrenergic receptors. Blocking these inhibits the sympathetic response. However, when abruptly terminated, they cause a reflexive opposite response. This patient will demonstrate increased contractility, blood pressure (BP), and heart rage (HR) as a result.

How well did you know this?
1
Not at all
2
3
4
5
Perfectly
5
Q

A patient has persistent midfoot pain with weight bearing. The injury occurred during a soccer match when an opposing plaver stepped on the patient’s right foot when it was planted and cutting to the left. Patient locates the pain where laces are tied. Upon examination there is splaving of the first metatarsal and increased pain when passively stressing the foot with plantarflexion and rotation. What injury should the therapist suspect the patient has sustained?
A. Lisfranc injury
B. Turf toe
C. Calcaneocuboid joint subluxation
D. Hallux rigidus

A

Correct Answer: A
The Lisfranc injury (also known as the Lisfranc fracture, tarsometatarsal injury, or simply midfoot injury) is an injury of the foot in which one or all of the metatarsal bones are displaced from the tarsus. Direct Lisfranc injuries are usually caused by a crush injury, such as when a heavy obiect falls onto the midfoot, or when landing on the foot after a fall from a significant height. The injury often occurs when an athlete has his or her foot plantar flexed and another player lands on his or her heel.

How well did you know this?
1
Not at all
2
3
4
5
Perfectly
6
Q

A patient is referred to physical therapy with a 10-vear history of rheumatoid arthritis (RA).
What are possible extra-articular complications?
A. Disc degeneration
B. Psoriatic skin and nail changes
C. Vasculitis
D. Conjunctivitis and iritis

A

Correct Answer: C
Rheumatoid arthritis is a progressive autoimmune disease affecting primarily joints and synovial tissue. Extra-articular complications of the disease can include vasculitis.

How well did you know this?
1
Not at all
2
3
4
5
Perfectly
7
Q

A physical therapist is instructing an elderly patient how to perform bed mobility following a total hip replacement. The therapist should carefully consider the effects of aging that relate to skin. What is one such effect?
A. Increased perception of pain
B. Impaired sensory integrity
C. Increased skin elasticity
D. Increased inflammatory responsiveness

A

Correct Answer: B
Changes in skin composition associated with aging include decreased sensitivity to touch, decreased perception of pain and temperature, and increased risk of injury.

How well did you know this?
1
Not at all
2
3
4
5
Perfectly
8
Q

What would a therapist who is examining the breathing pattern of a patient with a complete (ASIA A) C5 spinal cord injury expect to observe?
A. Asymmetric lateral costal expansion due to ASIA A injury.
B. An increased subcostal angle due to air trapping from muscle weakness.
C. No diaphragmatic motion since the diaphragm is below the level of the lesion.
D. Rising of the abdomen due to no abdominal muscle tone on the abdominal viscera.

A

Correct Answer: D
The abdominal musculature provides external stability to the abdominal viscera. Without this, the viscera are displaced with respiration.

How well did you know this?
1
Not at all
2
3
4
5
Perfectly
9
Q

Men are at high risk for development of metabolic syndrome if they exhibit which of the following symptoms?
A. An HDL level lower than 45mg/dL
B. A waist size greater than 40 inches
C. Triglyceride levels greater than 100mg/dL
D. Fasting blood glucose less than 100mg/dL

A

Correct Answer: B
Criteria for diagnosis of metabolic syndrome include abdominal obesity (waist circumference
>40 inches in men or > 35 inches in women).

How well did you know this?
1
Not at all
2
3
4
5
Perfectly
10
Q

During an examination, the limitations of ultrasound imaging include which of the following?
A. Inability to clearly see cartilate in infants
B. Disruption of cardiac pacemakers
C. Difficulty penetrating bone and therefore visualizing internal structure of bones
D. Inability to give a clear picture of tendons and therefore diagnose tendon tears

A

Correct Answer: C
Ultrasound has difficulty penetrating bone. Ultrasound images are typically used to help diagnose tendon tearsm such as tears of the rotator cuff in the shoulder or Achilles tendon in the ankle; abnormalities of the muscles such as tears; bleeding or other fluid collections within the muscles, bursae, and joints; benign and malignant soft tissue tumors; early changes of rheumatoid arthritis; fluid in a painful hip joint in children; lumbs in the neck muscles of infants; and soft tissue masses (lumps/bumps) in children.

How well did you know this?
1
Not at all
2
3
4
5
Perfectly
11
Q

A patient with a body mass index (BMI) of 37 is referred to physical therapy for exercise conditioning. What are additional clinical manifestations associated with the BMI that this patient might exhibit?
A. Hyperpnea and hyperpituitarism
B. Hypertension and hyperinsulinism
C. Hormone-related cancer
D. Hypolipoproteinemia and hypotension

A

Correct Answer: B
Obesity is associated with hypertension, dyslipidemia, hyperinsulinemia (type 2 diabetes), and hyperglycemia. The presence of these comorbidities increases risk, resulting in the need for additional medical screening before exercise testing.

How well did you know this?
1
Not at all
2
3
4
5
Perfectly
12
Q

When visually examining active abduction of the arm to 150 degrees, what is the normal composition of the motion a therapist would expect?
A. 150 degrees of the glenohumeral motion and O degrees of scapulothoracic motion
B. 110 degrees of the glenohumeral motion and 40 degrees of scapulothoracic motion
C. 75 degrees of the glenohumeral motion and 75 degrees of scapulothoracic motion
D. 100 degrees of the glenohumeral motion and 50 degrees of scapulothoracic motion

A

Correct Answer: D
Elevation of the arm into abduction involves coordinated motions involving the scapulothoracic and glenohumeral joints (along with movements at the sternoclavicular and acromioclavicular joints). While there is some variability in the precise timing and amounts during the early, middle, and late phases, motion at the glenohumeral and scapulothoracic joints generally occurs at an overall 2 to 1 ratio. Given this ratio, 150 degrees of active abduction would be a result of 100 degrees of glenohumeral and 50 degrees of scapulothoracic motion.

How well did you know this?
1
Not at all
2
3
4
5
Perfectly
13
Q

13.To prepare a patient with a cauda equine lesion for ambulation with crutches, what upper-quadrant muscles would be the most important to strengthen?
A. Upper trapezius, rhomboids, and levator scapulae.
B. Deltoid, coracobrachialis, and brachialis.
C. Middle trapezius, serratus anterior, and triceps.
D. Lower trapezius, latissimus dorsi, and pectoralis major.

A

Correct Answer: D
The muscles needed for crutch use include the shoulder depressors and extensors along with
elbow extensors.

How well did you know this?
1
Not at all
2
3
4
5
Perfectly
14
Q

What will a patient with a significant right thoracic structural scoliosis demonstrate on examination?
A. Decreased breath sounds on the right
B. Decreased thoracic rib elevation on the right
C. Increased lateral costal expansion on the right
D. Shortened internal and external intercostals on the right

A

Correct Answer: C
With a right thoracic scoliosis, the convex side is on the right. This would allow for increased aeration and mobility on that side.

How well did you know this?
1
Not at all
2
3
4
5
Perfectly
15
Q

A therapist has been treating a patient over a period of 4 months for decreased shoulder elevation and a loss of external rotation. Recovery has been good; however, the patient still complains of being unable to reach the upper shelves of kitchen cabinets and closets. To help the patient achieve this goal, what should be the focus of manual therapy?
A. Superior glide
B. Inferior glide
C. Anterior glide
D. Grade II oscillations

A

Correct Answer: C
Anterior glide would help increase external rotation (ER), which is a component of full elevation. Performing anterior glides to improve ER and late flexion will help increase overhead reach since ER of humerus occurs with flexion.

How well did you know this?
1
Not at all
2
3
4
5
Perfectly
16
Q

Which activity would help break up obligatory lower extremity synergy patterns in a patient with hemiplegia?
A. High kneeling position, ball throwing
B. Standing, alternate marching in place with hip and knee flexion and hip abduction
C. Sitting, alternate toe tapping
D. Sitting, foot slides under the seat

A

Correct Answer: A
Kneeling positions with the hip in extension and the knee flexed to 90 degrees, is an out-of-synergy position. Balance training activities (e.g., reaching, ball throwing) enhance postural control while engaging cognitive control on the added activity (ball throwing).

How well did you know this?
1
Not at all
2
3
4
5
Perfectly
17
Q

17.A patient recovering from stroke reports lack of feeling in the more-affected hand. Light touch testing reveals lack of ability to tell when the stimulus is being applied (only 1 correct response out of 10 tests). What addition sensory tests should the therapist perform?
A. Test for pain and temperature
C. Test for two-point discrimination
B. Test for two-point discrimination
D. Test for barognosis

A

Correct Answer: A
Testing for pain and temperature can be performed as these sensations are carried in different pathwavs (anterolateral spinothalamic pathways); light touch is carried in dorsal column-lemniscal pathways.

How well did you know this?
1
Not at all
2
3
4
5
Perfectly
18
Q

In managing the residual limb of an elderly patient with a transfemoral amputation, what is the MOST IMPORTANT factor the therapist should consider?
A. Contracture of hip musculature
B. Residual limb shape
C. Muscle atrophy
D. Residual limb healing

A

Correct Answer: D
All are important considerations. However, wound healing is most important to prevent infections and possible revision surgery or further amputation

How well did you know this?
1
Not at all
2
3
4
5
Perfectly
19
Q

Three months ago a patient experienced a traumatic injury to the hand that resulted in surgical tendon repair and fracture stabilization. The therapist is planning a treatment program to address tightness of the lumbricals. What exercises would be BEST in order to increase range of motion of the hand?
A. Both the MCP and IP joints are moved into flexion
B. Both the MCP and IP joints are moved into extension
C. The MCP joints are extended and the IP joints are flexed
D. The MCP joints are flexed and the IP joints are extended

A

Correct Answer: C
Lumbrical action is extension of the interphalangeal (P) joint and simultaneous flexion of the metacarpophalangeal (MCP) joint of the second through fifth digits. Therefore, the opposite motion of MCP joint extension and IP joint flexion would stretch the lumbricals.

How well did you know this?
1
Not at all
2
3
4
5
Perfectly
20
Q

20.A patient with a long history of cigarette smoking has been admitted to the hospital and presents with tachycardia, signs of lung infection, abnormal breath sounds in both lower lobes, and dullness to percussion. What should the therapist’s initial intervention focus on with this patient?
A. Getting the patient to quit smoking
B. Breathing reeducation to increase efficiency of ventilation
C. Airway clearance and secretion removal
D. Graded inspiratory muscle training

A

Correct Answer: C
The patient has signs and symptoms consistent with pneumonia. It is most important to assist with secretions clearance to assist with recovery from the infection and to improve gas exchange.

How well did you know this?
1
Not at all
2
3
4
5
Perfectly
21
Q

A patient has adhesive capsulitis of the glenohumeral joint. What is the expected greatest limitation of motion when performing shoulder ROM?
A. Flexion
B. Abduction
C. Medial rotation
D. Lateral rotation

A

Correct Answer: D
Adhesive capsulitis is diagnosed by numerous physical characteristics, including a thickening of the synovial capsule, adhesions within the subacromial or subdeltoid bursa, adhesions to the biceps tendon, and/or obliteration of the axillary fold secondary to adhesions. Adhesive capsulitis is commonly associated with other systemic and nonsystemic conditions. By far the most common is the comorbid condition of diabetes mellitus. The common capsular pattern of limitation has historically been described as diminishing motions with external (lateral) shoulder rotation being the most limited, followed closely by shoulder flexion and internal rotation.

How well did you know this?
1
Not at all
2
3
4
5
Perfectly
22
Q

During a home visit an adult patient asks the physical therapist assistant to see the physical therapy progress notes in the medical record. What should the physical therapist assistant do?
A. Refuse to let the patient see the record
B. Allow the patient to see the notes
C. Let the patient see the notes only with the permission of the physical therapist
D. Contact the patient’s physician and explain the situation

A

Correct Answer: B
The law requires a health care provider, except in limited circumstances, to supply a patient, upon request, complete and current information the provider has about the patient’s diagnosis, treatment, and prognosis. The provider must also notify a patient of any test results in the provider’s possession or requested by the provider for purposes of diagnosis, treatment, or prognosis.

How well did you know this?
1
Not at all
2
3
4
5
Perfectly
23
Q

A chest tube gets dislodged during physical therapy treatment. If the therapist fails to cover the defect, what could the patient develop?
A. Pulmonary embolism
B. Pulmonary edema
C. Pneumothorax
D. Aspiration pneumonia

A

Correct Answer: C
With the removal of the chest tube, there is an increased positive pressure on the lung tissue.
The lung is not able to inflate, and it succumbs to the pressure and therefore collapses.

How well did you know this?
1
Not at all
2
3
4
5
Perfectly
24
Q

Following a motor vehicle accident, a patient with chest trauma developed atelectasis. What is the LEAST Appropriate intervention to help with the immediate management of atelectasis?
A. Pain reduction techniques
B. Segmental breathing
C. Incentive spirometry
D. Paced breathing

A

Correct Answer: D
In order to reverse atelectasis, the patient needs a technique to facilitate deep breathing. Paced breathing controls the rate of breathing, not the depth of breathing, and will therefore be ineffective.

How well did you know this?
1
Not at all
2
3
4
5
Perfectly
25
Q

Following a reattachment of the flexor tendons of the fingers. the patient is in a splint. One physical therapy goal is to minimize adhesion formation. What should the physical theranist teach the patient to perform after 72 hours post-surgery?
A. Passive extension and active flexion of the interphalangeal joints
B. Active extension and flexion of the interphalangeal joints
C. Active extension and passive flexion of the interphalangeal joints
D. Gentle passive extension and flexion of the interphalangeal joints

A

Correct Answer: C
Severe edema increases tendon drag and likelihood of rupture. Therefore, wait until 48 to 72 hours postop prior to initiating range of motion (ROM therapy. This patient is a few days postop and can begin passive finger flexion with caution so as not to disrupt the repair. Begin by blocking the metacarpophalangeal (MCP) in full flexion and actively extend interphalangeal (IP) joints, followed by passive proximal interphalangeal (PIP) flexion and active extension.

How well did you know this?
1
Not at all
2
3
4
5
Perfectly
26
Q

26.A patient with a transtibial amputation of 2 months’ duration complains of an intense burning pain that seems to emanate from the heel. This phantom pain mirrors the patient’s preoperative pain. What is the most likely previous source of this pain?
A. Dorsalis pedis artery obstruction
B. Popliteal artery obstruction
C. Damage to the superficial peroneal (fibular) nerve
D. Damage to the tibial nerve

A

Correct Answer: D
Although the limb is no longer there, the nerve endings at the site of the amputation continue to send pain signals to the brain that make the brain think the limb is still there. Sometimes, the brain memory of pain is retained and is interpreted as pain regardless of signals from injured nerves. In addition to pain in the phantom limb, some people experience other sensations such as tingling, cramping, heat, and cold in the portion of the limb that was removed. The question describes pain in the heel. Nerve supply to the heel is by the calcaneal branch of the tibial nerve.

How well did you know this?
1
Not at all
2
3
4
5
Perfectly
27
Q

A patient’s plan of care includes use of iontophoresis for the management of calcific bursitis of the shoulder. To administer this treatment using the acetate ion, what current characteristics and polarity should be used?
A. Monophasic twin-peaked pulses using the positive pole
B. Monophasic twin-peaked pulses using the negative pole
C. Direct current using the positive pole
D. Direct current using the negative pole

A

Correct Answer: D
The acetate ion has a negative charge, and thus a negative pole will be needed to repel the drug into the tissue. Direct current will continuously drive the acetate into the tissue during the treatment time.

How well did you know this?
1
Not at all
2
3
4
5
Perfectly
28
Q

A snowmobile left the trail and struck a tree. The driver’s left knee was flexed approximately 90 degrees and the tibia impacted with the inside front of the snowmobile. What would this mechanism of injury MOST LIKELY result in?
A. Dislocated patella
B. Sprained or ruptured posterior cruciate ligament (PCL)
C. Sprained or ruptured anterior cruciate ligament (ACL)
D. Rupture of the popliteal artery

A

Correct Answer: B
The above scenario describes one of the most common mechanisms of injury of the PCL, the
“dashboard iniury.” This occurs when the knee is flexed, and an object forcefully strikes the proximal anterior tibia and displaces it posteriorly. It is called a “dashboard injury” because it can occur in automobile collisions when the tibia forcefully hits the dashboard. The PCL attaches from the lateral aspect of the medial femoral condyle to just posterior to the posterior horn of the medial meniscus. It is the primary restraint to posterior displacement of the tibia on the femur.

How well did you know this?
1
Not at all
2
3
4
5
Perfectly
29
Q

The interview with an 18 year-old female cross-country runner elicits a history of stiffness and diffuse ache in her right knee that is aggravated by prolonged sitting. Going down stairs is also painful. Based on this information, what is the LIKELY diagnosis that should serve as a focus for the physical examination?
A. Iliotibial band friction syndrome
B. Osgood-Schlatter disease
C. Meniscal tear
D. Patellofemoral syndrome

A

Correct Answer: D
Patellofemoral syndrome (“runner’s knee”) is the most common overuse iniury among runners.
It occurs due to mistracking of the patella within the intercondylar groove. It generallv occurs in younger, recreational runners and is more common in women The main symptom of patellofemoral pain syndrome is knee pain, especially when sitting with knee flexion. This is known as the theater sign or movie-goer’s knee. Squatting, jumping, or using the stairs (especially going down stairs) will also be painful. There may also be occasional buckling of the knee.

How well did you know this?
1
Not at all
2
3
4
5
Perfectly
30
Q

Following mastectomy with axillary Ivmph node dissection, a patient developed 4+ edema in the ipsilateral arm. A compression garment was ordered. What is the primary reason this garment decreases edema?
A. It decreases the osmotic pressure of the capillaries
B. It increases the capillary permeability
C. It exceeds the internal tissue hydrostatic pressure
D. It equals the fluid outflow from the capillaries

A

Correct Answer: C
The external pressure caused by the compression garment essentially increases the amount of pressure on the tissue. This causes a relative increase in the hydrostatic pressure in the extravascular space compared with the intravascular space.

How well did you know this?
1
Not at all
2
3
4
5
Perfectly
31
Q

A patient presents with a chronic restriction of the temporomandibular joint (TMJ). The physical therapist observes the situation seen in the picture during mouth-opening range of motion (ROM) assessment. What is the BEST intervention if the patient has a classic TMJ unilateral capsular restriction?
A. Left TMJ, superior glide manipulation
B. Left TMJ, inferior glide manipulation
C. Right TMJ, superior glide manipulation
D. Right TMJ, inferior glide manipulation

A

Correct Answer: D
Right TMJ, inferior glide. In the photo, the chin has deviated to the right at terminal opening.
The active range of motion (AROM) will be limited with ipsilateral opening and a lateral deviation to the side of restriction for patients with a TMJ capsular pattern of restriction.

How well did you know this?
1
Not at all
2
3
4
5
Perfectly
32
Q

A patient in the late stages of Parkinson’s disease exhibits episodes of akinesia while walking. What should the therapist examine?
A. Primary involvement of the head and trunk
B. Associated dyskinesias
C. Primary involvement of the hips and knees
D. Triggers that precipitate the freezing episodes.

A

Correct Answer: D
Freezing of gait (episodes of akinesia) is typically associated with a trigger (e.g., turning, changing direction or speed, doorways). Identification of triggers is helpful in developing the plan of care.

How well did you know this?
1
Not at all
2
3
4
5
Perfectly
33
Q

A patient is referred to physical therapy for balance and gait training following two falls in the home in the past month. The therapist notes in the medical record that the patient has adrenal insufficiency. What are the metabolic abnormalities associated with adrenal insufficiency?
A. Hypokalemia
B. Hyponatremia
C. Hyperglycemia
D. Alkalosis

A

Correct Answer: B
Metabolic abnormalities seen in adrenal insufficiency include hyponatremia (decreased sodium concentration in the blood) secondary to renal loss of sodium ions. A decrease in cortisol results in an inability to regulate potassium and sodium.

How well did you know this?
1
Not at all
2
3
4
5
Perfectly
34
Q

A physical therapist is treating a patient with active infectious hepatitis B. In addition to wearing a protective gown when in the patient’s room, what precautions should be taken to avoid transmission of the disease?
A. Avoid direct contact with the patient’s blood or blood-contaminated equipment by wearing gloves
B. Avoid direct contact with any part of the patient
C. Have the patient wear a mask to minimize droplet spread of the organisms from coughing
D. Provide tissues and no-touch receptacles for disposal of tissues

A

Correct Answer: A
Hepatitis B is transmitted in blood, body fluids, or body tissues. Precautions should include avoiding direct contact with blood or blood-contaminated equipment.

How well did you know this?
1
Not at all
2
3
4
5
Perfectly
35
Q

Idiopathic scoliosis is suspected in a 12-vear-old girl. During the physical examination, what is the standard screening test for this condition?
A. Longsitting, forward bend test
B. Standing, Adam’s forward bend test
C. Sitting, rotation test to the right and left
D. Standing, backward extension test

A

Correct Answer: B
Screening is most commonly done on adolescents. Females achieve adolescence about two years before males and are afflicted with scoliosis requiring treatment three to four times more frequently than males. The adam’s forward bend test is the standard screening test for scoliosis. During the test, the child will bend forward with feet together, knees straight, and arms hanging free. The therapist observes child from the back. Looking for a difference in the shape of the ribs on each side. A spinal deformity is most noticeable in this position.

How well did you know this?
1
Not at all
2
3
4
5
Perfectly
36
Q

A patient had anterior cruciate ligament (ACL) reconstructive surgery 2 weeks ago. During the initial examination, the physical therapist noticed marked edema around the knee and calf Knee passive range of motion (PROM was limited form -5 degrees extension to 90 degrees of flexion. Hip PROM was within normal limits. Ankle dorsiflexion was limited to 0 degrees because of pulling pain in the calf. Marked tenderness to superficial palpation to anterior knee and posterior calf and increased temperature were also noted. The patient is experiencing calf pain and discomfort when standing during partial weight-bearing ambulation. Upon completion of the examination, what is the BEST intervention at this time?
A. Isometrics and PROM for the knee
B. Massage to knee and calf to help alleviate the expected postsurgical edema
C. Ice and interferential current to alleviate edema and facilitate movement
D. Immediate referral to the surgeon

A

Correct Answer: D
The patient may have a deep vein thrombosis (DVT) based on increased swelling, temperature, tenderness, and pain in the calf with weight bearing 2 weeks after surgery, so referral to the surgeon is the best intervention at this time.

How well did you know this?
1
Not at all
2
3
4
5
Perfectly
37
Q

Use of continuous ultrasound at 1.5 watts/cm2 can increase which of the following?
A. Local metabolic rate
B. Rate of muscle hypertrophy
C. Stiffness of collagen tissue
D. The resolution of acute inflammation

A

Correct Answer: A
Continuous ultrasound (US) is used for its thermal effects, and the higher the intensity, the greater the probability of causing a tissue temperature rise that would result in an increased metabolic rate in the area applied.

How well did you know this?
1
Not at all
2
3
4
5
Perfectly
38
Q

During observation of bilateral active straight leg raising in a supine position, the patient demonstrates progressively increasing lumbar lordosis during lowering of the limbs with each successive lift. What is the MOST LIKELY cause of the observed excessive lordosis during the bilateral straight leg activity?
A. Muscle imbalance between the rectus femoris and the Sartorius muscles.
B. Weakness of both quadratus lumborum muscles.
C. Fatigue weakness of the rectus abdominis and oblique muscle groups
D. Excessive elastic shortening of the ipsilateral hamstring muscle group

A

Correct Answer: C
During a dynamic activity such as the performance of a bilateral active straight leg raise, the weight of the limbs lifted produces an anterior torque on the pelvis. During this dynamic activity, excessive lordosis typically is associated with excessive anterior pelvic rotation.
Normally muscles that control anterior pelvic rotation (posterior pelvic rotators) act to counter the anterior torques produced by the mass of the lower limbs lifted off the table, helping to prevent unwanted sagittal plane movements of the lumbar spine. Contractile activities of the anterior trunk muscles (rectus and obliques) provide a posterior rotation movement on the pelvis, helping to stabilize the pelvis. Fatigue weakness of the anterior trunk muscles in this patient could result in poor control of sagittal plane rotation of the pelvis, leading to the observed increasing lumbar lordosis.

How well did you know this?
1
Not at all
2
3
4
5
Perfectly
39
Q

The therapist is treating a patient with chronic Lyme disease of more than I vear’s duration.
What joints are likely to demonstrate more arthritic changes and therefore should be the focus of physical therapy interventions?
A. Small joints of the hands and feet
B. Large joints of the body, especially the knee
C. Axial joints, especially the lumbosacral spine
D. Axial joints, especially the cervical and thoracic spine

A

Correct Answer: B
Stage 3 Lyme disease (late or chronic Lvme disease is characterized by intermittent arthritis with marked pain and swelling, especially in thelarge joints) Permanent joint damage can occur.

How well did you know this?
1
Not at all
2
3
4
5
Perfectly
40
Q

During auscultation of the heart, the therapist hears S1 and S2 heart sounds. During early diastole the therapist also hears a low frequency sound of turbulence. What suspected abnormal sound should the therapist record this as?
A. S4 sound
B. S3 sound
C. Heart murmur
D. Pericardial friction rub

A

Correct Answer: B
S3 is an abnormal third heart sound due to poor ventricular compliance and turbulence, It is heard as a low frequency sound during early diastole.

How well did you know this?
1
Not at all
2
3
4
5
Perfectly
41
Q

A physical therapist observes a full-term infant in the neonatal intensive care unit (NICU) just after birth. In the supine position, the shoulders are abducted and externally rotated, elbows and fingers are flexed, hips are abducted and externally rotated, and knees are flexed.
What would this posturing be an indication of?
A. Upper extremity tone is abnormal
B. Lower extremity tone is abnormal
C. Tone is abnormal in both upper and lower extremities
D. Tone is normal in both upper and lower extremities

A

Correct Answer: D
A full-term infant in the NICU can have low Apgar scores, respiratory distress, or any one of a number of specific diagnoses (none listed in this case). Initial tone and posturing involve some flexion of the limbs. At 1 month, decreased flexion can be expected.

How well did you know this?
1
Not at all
2
3
4
5
Perfectly
42
Q

A patient with type 1 diabetes mellitus has generalized osteoporosis. What is the BEST exercise to include in this patient’s plan of care?
A. Bilateral quadriceps presses against resistance in sitting
B. Aquatic exercises
C. Running on a treadmill
D. Partial squats in standing

A

Correct Answer: D
Extensor stabilization exercises in weight-bearing postures provide the best stimulus to bone (e.g., standing, holding against resistance, standing partial squats).

How well did you know this?
1
Not at all
2
3
4
5
Perfectly
43
Q

A patient suddenly falls and lands on a piece of equipment left on the floor. A severe laceration with spurting blood is noted in the area of the lateral distal right thigh. To help control bleeding, where should the physical therapist apply pressure in addition to directly over the wound?
A. Behind the knee at the popliteal fossa
B. At the femoral triangle
C. At the antecubital fossa
D. At mid-thigh, directly over the profunda femoris artery

A

Correct Answer: B
This patient is experiencing an arterial bleed most likely affecting the descending branch of the lateral circumflex femoral artery. Pressure should be applied over the wound and more proximally over the femoral artery, which is palpable in the femoral triangle just inferior to the inguinal ligament.

How well did you know this?
1
Not at all
2
3
4
5
Perfectly
44
Q

While gait training a patient following a stroke, the therapist observes the knee on the hemiparetic side going into recurvatum during stance phase. What is the MOST LIKELY cause of this deviation?

A. Severe spasticity of the hamstrings or weakness of the gastrocnemius-soleus
B. Weakness or severe spasticity of the quadriceps
C. Weakness of the gastrocnemius-soleus or spasticity of the pretibial muscles
D. Weakness of both the gastrocnemius-soleus and pretibial muscles

A

Correct Answer: B
Weakness of severe spasticity of the quadriceps is the most likely cause of genu recurvatum.

How well did you know this?
1
Not at all
2
3
4
5
Perfectly
45
Q

This picture depicts a clinician assessing for Stemmer’s sign. The clinician is examining for what condition?
A. Bunion
B.Hammer toe
C. Lymphedema
D.Fracture of the second toe

A

Correct Answer: C
Stemmer’s sign is assessed by pulling up on the skin at the base of the second toe or finger, which the clinician is doing in this picture. If the skin is unable to be pulled up, then it is a sign of lymphedema, usually primary but also advanced secondary.

How well did you know this?
1
Not at all
2
3
4
5
Perfectly
46
Q

A child with spastic diplegia is becoming independent in using a walker. Additional goals desired by the physical therapist include improving posture and increasing the child’s energy efficiency and velocity while ambulating. In this case, which walker is most likely to help improve these goals?
A. A standard anterior walker with no wheels
B. An anterior rollator walker with two wheels
C. A posterior rollator walker with two wheels
D. A posterior rollator walker with four wheels

A

Correct Answer: D
A posterior rollator walker with four wheels is the best choice to improve upright posture, energy efficiency, and velocity.

How well did you know this?
1
Not at all
2
3
4
5
Perfectly
47
Q

What is pain and tenderness with palpation over McBurney’s point associated with?
A. Acute appendicitis
B. Hiatal hernia
C. Acute cholecystitis
D. GERD

A

Correct Answer: A
Pain and tenderness with palpation over McBurney’s point are associated with acute appendicitis. McBurney’s point is located half the distance between the anterior superior iliac spine (ASIS) and the umbilicus in the right lower abdominal quadrant.

How well did you know this?
1
Not at all
2
3
4
5
Perfectly
48
Q

What is the expected hemody namic response for a patient on a beta-adrenergic blocking agent during exercise?
A. Heart rate to be low at rest and rise minimally with exercise
B. Heart rate to be low at rest and rise continuously to expected levels as exercise intensity
increases
C. Systolic blood pressure to be low at rest and not rise with exercise
D. Systolic blood pressure to be within normal limits at rest and progressively fall as exercise intensity increases.

A

Correct Answer: A
A beta-blocker will decrease the sympathetic response to activity. This will decrease the heart rate at rest and will blunt the heart rate response to activity.

How well did you know this?
1
Not at all
2
3
4
5
Perfectly
49
Q

A neonate’s Apgar score at 1 minute after birth is 8; at 5 minutes it is 9. Based on this score and expected heart rate, what can the therapist conclude about this infant?
A. The infant would have a heart rate of less than 100 beats per minute with slow and irregular respirations and not require resuscitation
B. The infant would require extensive resuscitation efforts including intubation
C. The infant would require some resuscitation and administration of supplemental oxygen
D. The infant would have a heart rate of greater than 100 beats per minute with good respiration and not require resuscitation

A

Correct Answer: D
Early signs receiving the top Apgar score of 2 each include heart rate over 100 bpm, good respiration and cring, active movements, cough or sneeze, and pink color (total of 10). Apgar scores of 8 to 10 at 1 minute are considered normal.

How well did you know this?
1
Not at all
2
3
4
5
Perfectly
50
Q

With the patient supine, the vertebral artery test is performed by passively moving the head and neck into extension and side flexion, then rotation to the same side and holding for 30 seconds. Which of the following indicates a positive test?
A. Sensory changes occur in the face along with visual changes
B. Dizziness or nystagmus occurs, indicating that the opposite side artery is being compressed
C. Hearing difficulties and facial paralysis occur
D. Dizziness or nystagmus occurs, indicating that the same side artery is being compressed

A

Correct Answer: B
The test is positive if dizziness or nvstagmus occurs, indicating that the opposite side artery is affected. Current evidence demonstrates some limitations in use of this test; however, not performing it prior to cervical manual therapy could be considered a breach of the standard of care.

How well did you know this?
1
Not at all
2
3
4
5
Perfectly
51
Q

A young adult who is comatose (Glasgow Coma Scale score of 3) is transferred to a long-term care facility for custodial care. On initial examination, the therapist determines the patient is demonstrating decerebrate posturing. Which limb or body position is indicative of this?
A. The upper extremities in flexion and the lower extremities in extension
B. Extreme hyperextension of the neck and spine with both lower extremities flexed and the heels touching the buttocks
C. All four limbs in extension
D. All four limbs in flexion

A

Correct Answer: C
With decerebrate posturing (decerebrate rigidity), the upper and lower extremities are held rigidly in extension.

How well did you know this?
1
Not at all
2
3
4
5
Perfectly
52
Q

A patient is recovering from a mild stroke with trunk weakness and postural instability. The patient complains of severe heartburn. What is the BEST choice to maximize stroke recovery and improve trunk stabilization while minimizing heartburn?
A. Perform trunk stabilization exercises with the patient in the semi-Fowler position
B. Begin with bridging exercises progressing to sitting holding
C. Perform resisted holding in sitting using rhythmic stabilization
D. Take antacids before physical therapy

A

Correct Answer: C
Heartburn is a common symptom of gastrosophageal reflux disease (GERD) and can be aggravated by positioning in supine, prone, or bridging. Modifying the patient’s position to upright can alleviate the symptoms and demonstrate to the patient the therapist’s concern.

How well did you know this?
1
Not at all
2
3
4
5
Perfectly
53
Q

To prevent contractures in a newly admitted patient with anterior neck burns, it would be best to position the neck in which of the following’
A. Hyperflexion
B. Slight flexion
C. Neutral
D. Slight extension

A

Correct Answer: D
Positioning the head/neck in slight extension will counteract the expected pull of the head into flexion with contracture development.

How well did you know this?
1
Not at all
2
3
4
5
Perfectly
54
Q

A therapist wishes to examine the balance of an elderly patient with a history of falls. The
Berg Balance Test is selected. Which area isNOT examined using this test?
A. Sit-to-stand transitions
B. Functional reach in standing
C. Turning while walking
D. Tandem standing

A

Correct Answer: C
The Berg Balance Test (BBT is a test of static and dynamic balance in sitting and standing, includes transitional items of sit-to-stand and stand-to-sit. It does not include items or
gait. Turning while walking is an item on both the Tinetti Performance-Oriented Mobility
Assessment and the Dynamic Gait Index.

How well did you know this?
1
Not at all
2
3
4
5
Perfectly
55
Q

The physical therapist is instructing a new mother to perform range of motion and stretching for her newborn who has a clubfoot. In what directions should the therapist advise her to carefully stretch?
A. Plantarflexion and inversion
B. Plantarflexion and eversion
C. Dorsiflexion and inversion
D. Dorsiflexion and eversion

A

Correct Answer: D
The term “clubfoot” (talipes equinovarus) refers to the way the foot is positioned at a sharp angle to the ankle, like the head of a golf club. It describes a range of foot abnormalities usually present at birth in which the infant’s foot is twisted into an equinovarus deformity. Stretching should be opposite the direction of the deforming position; therefore, stretch is into dorsiflexion and eversion.

How well did you know this?
1
Not at all
2
3
4
5
Perfectly
56
Q

Pursed lip breathing as part of the treatment regimen would be MOST appropriate for a patient with which condition?
A. Circumferential thoracic burns
B. Asbestosis
C. Rib fracture
D. Emphysema

A

Correct Answer: D
Pursed lip breathing gives increased resistance to the airwavs on exhalation. The resistance causes increased pressure, which helps to prevent airway collapse (likely sequelae given the pathophysiology of emphysema). This occurs via collateral ventilation through pores of Kohn and canals of Lambert.

How well did you know this?
1
Not at all
2
3
4
5
Perfectly
57
Q

A patient has normal quadriceps strength but unilateral weakness (3/5) of the hamstring muscles on the right. What might the therapist observe during swing phase of gait?
A. Excessive compensatory hip extension on the sound side
B. Decreased hip flexion followed by increased knee flexion on the weak side
C. Excessive hip extension followed by abrupt knee extension on the weak side
D. Excessive hip flexion followed by abrupt knee extension on the weak side

A

Correct Answer: D
The hamstring muscles primarily control the forward swing of the leg during terminal swing.
Loss of function may result in abrupt knee extension and increased hip flexion.

How well did you know this?
1
Not at all
2
3
4
5
Perfectly
58
Q

Following cast immobilization for a now healed supracondvlar fracture of the humerus, a patient’s elbow lacks mobility. To increase elbow range of motion, joint mobilization in the maximum loose-packed position should be performed at what position?
A. Full extension
B. 90 degrees of flexion
C. 70 degrees of flexion
D. 30 degrees of flexion

A

Correct Answer: C
The loose-packed position of the humeroulnar ioint is 70 degrees of flexion. The loose-packed position for the radiohumeral oint is 70 degrees of flexion, 35 degrees of supination. The loose-packed position is basically the resting position where the joint capsule and ligaments are most relaxed.

How well did you know this?
1
Not at all
2
3
4
5
Perfectly
59
Q

A patient with a complete tetraplegia (ASIA A) at the C6 level is initially instructed to transfer using a transfer board. With shoulders externally rotated, how should the remaining upper extremity (UE) joints be positioned?
A. Forearms pronated with wrists and fingers extended
B. Forearms supinated with wrist extended and fingers flexed
C. Forearms pronated with wrists and fingers flexed
D. Forearms supinated with wrists and fingers extended

A

Correct Answer: B
The patient with tetraplegia at the C6 level does not have triceps to assist in transfers.
Independent transfers can be achieved using muscle substitution and positioning to lock the elbow. The hands are positioned anterior to the hips; the shoulders are externally rotated with elbows and wrists extended, forearms supinated, and fingers flexed. Strong contraction of the anterior deltoid, shoulder external rotators, and clavicular portion of the pectoralis major flexes and adducts the humerus, causing the elbow to extend.

How well did you know this?
1
Not at all
2
3
4
5
Perfectly
60
Q

Upon removing the dressing covering a decubitus ulcer located on the heel of an elderly patient, the physical therapist observes copious amounts of a foul-smelling, yellow-green discharge. How should the therapist document this finding in the patient’s medical record?
A. There is likelihood of a staphylococcus aureus infection
C. Wound exudate is purulent
B. Maceration of the wound is evident
D. Wound exudate is serosanguinous

A

Correct Answer: C
Wound exudate (drainage) is purulent (containing pus) on the basis of the yellow-green discharge.

How well did you know this?
1
Not at all
2
3
4
5
Perfectly
61
Q

During examination of the right shoulder of a teenager with anterior shoulder pain, the physical theranist notices an excessive amount of scapular abduction during both shoulder flexion and abduction. Full range of glenohumeral (GH) motion is achieved at the ends flexion and abduction. The axillary border of the scapula protrudes laterally beyond the thorax much more on the right as compared to the left. Which muscle(s) would be associated with excessive lengthening during the movements of shoulder flexion and abduction?
A. Serratus anterior
B. Rhomboids
C. Teres major
D. Levator scapula

A

Correct Answer: B
During the motions of flexion and abduction, concentric muscle activity of the serratus anterior occurs, providing upward scapular rotation. However, concentric activity of the serratus anterior would also induce abduction of the scapula due to its line of action. Excessive scapular abduction can be controlled through muscle actions from those that can provide scapular adduction. The primary scapular adductors acting eccentrically to control excessive scapular abduction include the rhomboid and the trapezius muscle groups. Inadequately controlled lengthening of the rhomboids could contribute to hyperabduction of the scapula during the mid and later phases of shoulder flexion and abduction.

How well did you know this?
1
Not at all
2
3
4
5
Perfectly
62
Q

A competitive gymnast is examined by the physical therapist. The chief complaint is nagging, localized pain in the anterior left lower leg that is consistently present at night and increases during activity with swelling. What are these complaints MOST characteristic of?
A. Bone tumor
B. Anterior compartment syndrome
C. Shin splints
D. Stress fracture

A

Correct Answer: D
Symptoms of a stress fracture may include pain and swelling, particularly with weight bearing on the injured bone. Stress fractures should be considered in patients who present with tenderness or edema after a recent increase in activity or repeated activity with limited rest. The differential diagnosis varies based on location but commonly includes tendinopathy, compartment syndrome, and nerve or artery entrapment syndrome.

How well did you know this?
1
Not at all
2
3
4
5
Perfectly
63
Q

Four days following open-heart surgery, a patient is ambulating with a physical therapist in the hallway. The patient complains of some chest discomfort during the activity and wishes to return to his or her room. What should the therapist do?
A. Sit the patient down and call the physician immediately
B. Complete the treatment and have an aide transport the patient to the room as some discomfort is expected
C. Call the nurse and check to see if the discomfort is ongoing
D. Sit the patient down, take vital signs, and inform nursing services of the patient’s complaint

A

Correct Answer: D
It is important to determine etiology of the pain. If it is angina. having the patient stop will reduce the myocardial demand. Gathering timely vital signs will help determine if the patient was maintaining his or her cardiac output. If not, the demand of the activity may have been too great.

How well did you know this?
1
Not at all
2
3
4
5
Perfectly
64
Q

A group of institutionalized elderly was examined for balance instability and fall risk using a standardized test, the Performance-Oriented Mobility Assessment (POMA). The test-retest reliability fo total test (POMA-T) and the subtests, balance subtest (POMA-B) and gait subtest (POMA-G) varied between 0.77 and 0.86 the interrater reliability values ranged from 0.80 to 0.93. what is the therapist’s correct interpretation of these findings?
A. Overall, the test demonstrated moderate reliability
B. The test demonstrated moderate reliability for test-retest and good reliability for interrated comparisons
C. The test demonstrated poor reliability for test-retest and moderate reliability for interrater comparisons
D. Overall, the test demonstrated good reliability

A

Correct Answer: D
Reliability coefficients above 0.75 lemonstrate good reliability. This test demonstrated good reliability both for test-retest and interrater comparisons.

How well did you know this?
1
Not at all
2
3
4
5
Perfectly
65
Q

A patient presents with hemosiderin changes and increased lower extremity edema. What diagnosis are these changes consistent with?
A. Chronic venous insufficiency
B. Acute venous insufficiency
C. Acute arterial insufficiency
D. Chronic arterial insufficiency

A

Correct Answer: A
Lower extremity edema is usually due to incompetent valves. which causes the edema. Longstanding edema causes staining of the legs because of increased iron from pooling blood.

How well did you know this?
1
Not at all
2
3
4
5
Perfectly
66
Q

A patient experienced a cerebrovascular accident (right CA) 2 weeks ago. The patient has motor and sensory impairments primarily in the left lower extremity; the left upper extremity shows only mild impairment. There is some confusion and perseveration. Based on these findings, what type of stroke syndrome does this patient present with?
A. Posterior cerebral artery stroke
B. Internal carotid syndrome
C. Anterior cerebral artery syndrome
D. Middle cerebral artery syndrome

A

Correct Answer: C
These signs and symptoms are characteristic of anterior cerebral artery (ACA) svndrome, with contralateral hemiplegia and lower extremities more affected than upper extremities.

How well did you know this?
1
Not at all
2
3
4
5
Perfectly
67
Q

A patient is walking on a motorized treadmill and is undergoing ECG monitoring. Based on viewing this cardiac rhythm strip, what action should the physical therapist take?
A. Immediately call emergency medical services
B. Modify the exercise based on past cardiac disease
C. Modify the exercise based on current cardiac disease
D. Exercise the patient/client without any cardiac restrictions

A

Correct Answer: D
This is sinus bradycardia. There are no restrictions on proceeding with exercise in this patient.

How well did you know this?
1
Not at all
2
3
4
5
Perfectly
68
Q

A therapist is planning to use percussion and shaking for assisting airway clearance with a patient diagnosed with chronic obstructive pulmonary disease (COPD). What major precaution might curtail selection of this form of intervention?
A. A platelet count of 30,000
B. Dyspnea when in the Trendelenburg position
C. Sa02 range of 88% to 94% on room air
D. Functional Independence Measure (FIM) score of 4

A

Correct Answer: A
A patient with a platelet count of 30,000 is at increased risk for bleeding. Percussion may cause microtraumas and increased bleeding risk.

How well did you know this?
1
Not at all
2
3
4
5
Perfectly
69
Q

A patient with degenerative joint disease of the right hip complains of pain in the anterior hip and groin, which is aggravated by weight bearing. There is decreased range of motion and capsular restrictions. Right gluteus medius weakness is evident during ambulation, and there is decreased tolerance of functional activities including transfers and lower extremity dressing. In this case, a capsular pattern of joint motion should be evident by which of the following?
A. Hip flexion, abduction, and internal rotation
B. Hip flexion, adduction, and internal rotation
C. Hip extension, abduction, and external rotation
D. Hip flexion, abduction, and external rotation

A

Correct Answer: A
The two classical capsular patterns at the him are FAME and MEAL. FAME stands for loss of flexion, abduction, internal (medial) rotation, and extension. MEAL is loss of medial (internal) rotation, extension, and abduction. According to the American Physical Therapy Association (APTA) hip pain practice guidelines, patients were classified as having hip osteoarthritis (A) if they (1) reported experiencing hip pain and (2) present with either one of the following clusters of clinical findings: (a) hip internal rotation less than 15 degrees, along with hip flexion less than or equal to 115 degrees and age greater or equal to 15, along with pain with hip internal rotation, duration of morning stiffness of the hip less than or equal to 60 minutes, and age greater than 50 years.

How well did you know this?
1
Not at all
2
3
4
5
Perfectly
70
Q

Confirmation of a diagnosis of spondylolisthesis can be made when viewing an oblique radiograph of the spine, What is the relevant diagnostic finding?
A. Posterior displacement of L5 over S1
B. Bamboo appearance of the spine
C. Compression of the vertebral bodies of L5 and S1
D. Bilateral pars interarticularis defects

A

Correct Answer: D
Spondvlolisthesis is defined as forward translation of a vertebral body with respect to the vertebra below. Spondvlolvsis, a break in the vertebra typically in the region of the pars interarticularis, may or may not be associated with a spondylolisthesis. If the pars defect is bilateral, it may allow slippage of the vertebra, typically L5 on S1, resulting in spondylolisthesis. Most cases are thought to result from minor overuse trauma, particularly repetitive hyperextension of the lumbar spine.

How well did you know this?
1
Not at all
2
3
4
5
Perfectly
71
Q

A physical therapist and physical therapist assistant are conducting a cardiac rehabilitation session for 20 patients. The therapist is suddenly called out of the room. The physical therapist assistant should do which of the following?
A. Terminate the exercises and have the patients monitor their pulses until the therapist
returns
B. Have the patients continue with the same exercise until the therapist returns
C. Have the patients switch to a less intense exercise until the therapist returns
D. Continue with the outlined exercise progression for that session

A

Correct Answer: D
The physical therapist provided an exercise program, and it is appropriate for the PTA to continue to follow it.

How well did you know this?
1
Not at all
2
3
4
5
Perfectly
72
Q

A physical therapist is examining a patient who has a recent history of falls while ambulating on level surfaces. Interaction with the patient indicates that cognition is unaffected. Which tests should be performed next once it has been established that cognition is not impaired?
A. Static balancing tests
B. Locomotor tests
C. Sensory testing
D. Dynamic balance tests

A

Correct Answer: C
Sensory testing should be performed next, including somatosensations from the feet and ankles, visions, and visual proprioception, and vestibular sensations. Once the primary sensations have been tested, the clinical Test for Sensory Interaction in Balance can reveal valuable information about the patient’s ability to use sensory information under changing balance conditions.

How well did you know this?
1
Not at all
2
3
4
5
Perfectly
73
Q

A therapist wishes to study the progress of patients with paraplegia who are discharged from a rehabilitation setting. Starting with the patient’s discharge, and once a month for 3 years, the therapist will measure their joint range of motion of both hips and knees. Accurate analysis of this time series study is heavily dependent upon which of the following?
A. Random sampling
B. Interrater reliability
C. Intrarater reliability
D. Predictive validity of the measurements

A

Correct Answer: C
Accurate analysis will depend on the therapist’s ability to consistently measure ROM using standardized techniques on multiple measurement trials (intrarater reliability).

How well did you know this?
1
Not at all
2
3
4
5
Perfectly
74
Q

Which of these findings is characteristic of a boutonniere deformity of the finger?
A. Flexion of the distal interphalangeal joint
B. Contracture of the extensor digitorum communis tendon
C. Rupture with volar slippage of the lateral bands
D. Hyperextension of the proximal interphalangeal joint

A

Correct Answer: C
Finger position marked by extension of the metacarpophalangeal (MCP) and distal interphalangeal (DIP) joints with flexion of the proximal interphalangeal (PIP) joints. Through injury or disease, the central extensor tendon (extensor digitorum communis [EDC]) of the involved finger ruptures. The tendon then displaces palmarly relative to the PIP joint (volar slippage). The lateral bands become tight and are attached to the distal phalanx, extending it.
The flexor digitorum profundus (FPD) is now unopposed, and it pulls the PIP joint into flexion.
The result is a deformity that looks like a “buttonhole” or boutonniere in French.

How well did you know this?
1
Not at all
2
3
4
5
Perfectly
75
Q

A patient with chronic asthma has been admitted to the hospital for an acute exacerbation.
What is the MOST important information the therapist needs in order to determine the patient’s prognosis with physical therapy?
A. A current medication list
B. A previous history of the disease
C. The most recent chest x-ray results
D. The most recent pulmonary function test results

A

Correct Answer: D
Recent pulmonary function test results will give the therapist information regarding the severity of the lung disease. This information will assist in determining how much the patient will progress.

How well did you know this?
1
Not at all
2
3
4
5
Perfectly
76
Q

A therapist is examining the gait of a patient with a transfemoral prosthesis. The patient circumducts the prosthetic limb during swing. The therapist needs to identify the cause of the gait deviation. What is the MOST likely prosthetic cause?
A. Unstable knee unit
B. Inadequate socket flexion
C. Sharp or high medial wall or abducted hip joint
D. Inadequate suspension or loose socket

A

Correct Answer: D
Prosthetic causes of circumduction include a long prosthesis, locked knee unit, loose knee friction, inadequate suspension, small or loose socket, and plantar flexed foot.

How well did you know this?
1
Not at all
2
3
4
5
Perfectly
77
Q

A patient diagnosed with multiple sclerosis experiences sudden electric-like shocks spreading down the body elicited by the maneuver in the picture. This is known as which of the following?
A. Head jolt test
B. Kernig’s sign
C. Lhermitte’s sign
D. Tinel’s test

A

Correct Answer: C
Chermitte’s sign is sudden, transient. electric-like shocks spreading down the body when the head is flexed forward. It occurs chiefly in patients with multiple sclerosis but can also be seen in compression disorders of the cervical spine (tumor, cervical spondylitic myelopathy).

How well did you know this?
1
Not at all
2
3
4
5
Perfectly
78
Q

An elderly patient with degenerative joint disease is seen by a physical therapist 3 days following a total knee replacement. Which of these findings would be an indication for the therapist to contact the surgeon?
A. Patient is noncompliant when learning to transfer properly
B. Patient cannot ambulate at least 50 feet with a standard walker
C. Patient fails to recognize the therapist on the third consecutive postoperative visit
D. Patient complains of soreness at the incision site

A

Correct Answer: C
Postoperative adverse effects on the cardiac, pulmonary, and neuromuscular systems and on cognitive function are the main concerns for elderly surgical patient who are at high risk.
Postoperative delirium is characterized by incoherent thought and speech, disorientation, impaired memory, and attention. Elderly patients usually manifest delirium following a lucid interval of 1 postoperative day or more, a condition known as interval delirium, Symptoms are often worse at night. Alternatively, the condition can be silent and unnoticed, or misdiagnosed as depression. However, the effects of elderly postoperative delirium are evident in increased morbidity, delayed functional recovery, and prolonged hospital stay. Fortunately, the postoperative cognitive dysfunction is a reversible condition in the majority of elderly surgical patients. Preoperative risk factors of bilateral total knee arthroplasty are associated with a significantly higher incidence of acute delirium than unilateral total knee arthroplasty in patients over 80 vears.
Failing to recognize the therapist after three visits is an indication of a declining mental condition. This would definitely be a safety consideration as the patient may not be able to follow all the precautions and may also put himself or herself in danger by walking without an ambulatory aid, etc. Wound infection is also a consideration. Contacting the surgeon is necessary.

How well did you know this?
1
Not at all
2
3
4
5
Perfectly
79
Q

The primary contribution of a physical therapist member of a facility emergency/disaster preparedness committee in formulating a disaster plan is describing the role of the physical therapists in providing which of the following?
A. Triage and basic life support during the disaster
B. Evaluation of soft tissue injuries and rendering appropriate care
C. Unique preparedness concerns needed for people with disabilities or special needs
D. Emotional distress management of victims or patients during crisis situations

A

Correct Answer: C
The physical therapist’s unique contribution to the team is in addressing the concerns of individuals with disabilities or special needs.

How well did you know this?
1
Not at all
2
3
4
5
Perfectly
80
Q

Following a hard tackle, a football player exhibits signs of fractured ribs and a pneumothorax. When auscultating during inhalation over the injured area, what would the physical therapist expect to hear?
A. Soft, rustling sounds on inhalation
B. Decreased or no breath sounds crackles
C. Crackles
D. Wheezes

A

Correct Answer: B
The fractured ribs will cause the patient to have pain and therefore not take deep breaths. More importantly, the pneumothorax will cause an increasing positive pressure on the lung, not allowing it to inflate. The result will be minimal air movement and decreased or absent breath sounds.

How well did you know this?
1
Not at all
2
3
4
5
Perfectly
81
Q

a patient with cystic fibrosis (CF) has been admitted to the hospital in acute respiratory failure as a result of an infection. What is the BEST choice for use of airway clearance techniques?
A. Should not be administered since it is contraindicated in acute respiratory failure
B. Should be administered two times a day to the patient’s tolerance
C. Should be administered according to the patient’ current home regimen
D. Should be administered vigorously once every 2 hours

A

Correct Answer: D
Given the pathology of CF, it is important to clear all secretions as often as possible to assist with clearing the infection. It will also assist with maximizing gas exchange.

82
Q

While ambulating a patient in the parallel bars, a therapist loses control and the patient falls, hitting his or her head on the bar. The patient lies motionless on the floor between the bars bleeding heavilv from a scaln laceration. What is the first thing the therapist should do?
A. Apply a thick gauze and manual pressure to the scalp wound
B. Check for responsiveness
C. Call emergency medical services
D. Immediately determine the patient’s heart rate and blood pressure

A

Correct Answer: B
The physical therapist’s first course of action is to check for responsiveness.

83
Q

A therapist is examining a patient with an ulcer in the lower leo/ankle and suspects it is an arterial rather than a venous ulcer. One of the factors the therapist uses to determine this is based on the location of the ulcer. What is the typical location of an arterial ulcer?
A. Medial malleolus
B. Posterior tibial area
C. Lateral malleolus
D. Medial distal tibia

A

Correct Answer: C
The typical location of an Arterial ulcer is the distal lower leg (toes. foot), the lateral malleolus, or the anterior tibial area.

84
Q

During pregnancy, the presence of the hormone relaxin can lead to abnormal movement and pain. Which joints are typically affected?
A. Glenohumeral joints
B. Hip joints
C. Lumbrosacral joints
D. Sacroiliac joints

A

Correct Answer: D
The sacroiliac (SI) oints are most often affected in pregnancy, resulting in pain.

85
Q

A patient with angina pectoris has been instructed to use sublingual nitroglycerin in case of an acute angina attack. What are the primary effects of this medication?
A. Vasoconstriction of peripheral vessels
B. Vasodilation of the coronary vessels
C. Increasing myocardial oxygen consumption
D. Increasing left ventricular end-diastolic pressure

A

Correct Answer: B
Nitroglycerin increases coronary blood flow by dilating coronary arteries and improving flow to ischemic areas. In low doses it produces vasodilation (venous greater than arterial) and is used in the acute and long term prophylactic management of angina pectoris.

86
Q

A patient has a body mass index (BMI) of 32 kg/m? with excessive tissue mass in the hip area. What accommodations are needed to the wheelchair prescription for this patient?
A. Move the small front casters closer to the drive wheels to increase stability
B. Add friction rims to increase handgrip function
C. Add an antitipping device to prevent falls going up curbs
D. Displace the rear axle forward for more efficient arm push

A

Correct Answer: D
This patient is obese. A bariatric wheelchair with heav-duty, extra-wide wheels is necessary.
The rear axle is displaced forward compared to the standard wheelchair to allow for more efficient arm push.

87
Q

A patient is diagnosed with benign paroxysmal positional vertigo (BPPV). What intervention should the plan of care for this patient emphasize?
A. Gaze stability exercises using horizontal head rotation (X1 viewing)
B. Canalith repositioning treatment
C. Postural stability exercises in sitting using a therapy ball
D. Habituation exercises using provocative positions and movements

A

Correct Answer: B
The goal of treatment is to remove the otoconia that have become dislodged and are free-floating in the semicircular canal (SCC), or canalithiasis. The patient’s head is guided through a series of movements to move the debris out of the involved SCC and into the vestibule. Once moved, the symptoms should resolve. Canalith repositioning maneuver (modified Epley) is used for canalithiasis.

88
Q

Following a cerebral vascular accident (CA), a patient is hospitalized. The therapist determines that a positioning schedule should be implemented. While in supine, what is the BEST position for the patient’s affected upper extremity?
A. Shoulder protracted and slightly abducted with external rotation, elbow extension, wrist neutral, and fingers extended.
B. Shoulder protracted and slightly abducted with internal rotation, elbow flexion, wrist neutral and fingers extended
C. Shoulder retracted and adducted with internal rotation and elbow, wrist, and fingers flexed
D. Shoulder retracted and abducted with external rotation, elbow extension, wrist neutral and fingers flexed

A

Correct Answer: A
Positioning is opposite) the expected posturing and spastic patterns of the more affected upper extremity (UE). This includes shoulder protraction with abduction and external rotation, elbow extension, wrist neutral, and fingers in extension.

89
Q

What is an acceptable modified position to drain the posterior basal segment of the left lower lobe in a patient with pulmonary congestion?
A. Side-lying on the right, with a pillow under the right hip and the bed flat
B. Prone, with a pillow under the hips and the bed flat
C. Side-lying on the right, with a pillow between the legs and the foot of the bed elevated 18 inches
D. Prone, with a pillow under the hips and the bed elevated 18 inches

A

Correct Answer: B
Prone, with a pillow under the hips and the bed flat will raise the posterior basal segments up to facilitate drainage.

90
Q

When the ankle is forcibly inverted and plantar flexed, which ligament is MOST FREQUENTLY sprained?
A. Deltoid
B. Anterior talofibular
C. Posterior talofibular
D. Calcaneofibular

A

Correct Answer: B
The anterior talofibular ligament is the most frequently inured ligament in the ankle, which occurs with an inversion sprain.

91
Q

A patient complains of excessive upper and lower extremity muscle aching, cramping, and right upper quadrant pain when exercising. The patient has a history of chronic alcoholism and was placed on atorvastatin (a statin drug) 2 months ago. The therapist should refer the patient to the primary care physician for which reason?
A. For an exercise test to determine the right intensity for exercise
B. To rule out cirrhosis of the liver
C. To rule out liver and muscle dysfunction from statin
D. To rule out gallstones that may be obstructing the bile duct

A

Correct Answer: C
A small percentage of patients (<5%) who take statins (atorvastatin such as Lipitor, or others) can experience myalgia, cramps, stiffness, spasm, or weakness affecting exercise tolerance. The patient needs to see the primary care physician to have the dose or medication changed.

92
Q

Following a period of spinal shock, a patient with a complete spinal cord injury (ASIA A) at the T5 level is placed on a bladder training program coordinated by the nurse. A realistic ultimate outcome for this program would be independent voiding by using which of the following?
A. The Crede maneuver
B. The Valsalva maneuver
C. A timed voiding program
D. Suprapubic stroking or tapping

A

Correct Answer: D
A spastic or reflex (upper motor neuron [UMN]) bladder contracts and reflexively empties in response to a certain level of filling pressure. Reflex empting can be triggered by manual stimulation techniques (e.g., stroking, kneading, or tapping the suprapubic area.

93
Q

A physical therapist is performing an examination of an elderly patient who is confined to bed in a custodial care facility. A large ulcer is observed on the right heel as shown in the picture. Based on the staging of pressure ulcers, how would this ulcer be classified?
A. Stage I ulcer
B. Stage II ulcer
C. Stage III ulcer
D. Stage IV ulcer

A

Correct Answer: C
A stage III ulcer is characterized by full-thickness skin loss with damage to or necrosis of subcutaneous tissue. It presents clinically as a crater.

94
Q

How would the clinical status of a patient with a posterior herniated nucleus pulposus be determined if there is improvement?
A. Peripheral pain increases only when lumbar extension is attempted
B. Peripheral pain occurs only with straight leg raising
C. Pain centralizes with passive hyperextension of the spine
D. There is flattening of the lumbar lordosis

A

Correct Answer: C
Centralization describes the phenomenon hy which distal limb pain, coming from the spine although not necessarily felt there, is immediately or eventually abolished in response to loading strategies. During centralization, the response to therapeutic loading strategies is assessed. Pain is progressively abolished in a distal to proximal direction with each progressive movement until all symptoms are abolished. In back pain only, the pain moves from a widespread to a more central location and then abolishes. Centralization has been proved to be a good indicator of a positive outcome in patients with low back pain.

95
Q

The physical therapist is examining the muscle length of the patient’s left hip and knee. How should the therapist interpret the muscle length test shown in the picture?
A. Shortness of one joint and tow joint hip flexors
B. Shortness of one joint hip flexor with normal two joint hip flexors
C. Normal one joint hip flexors with tightness of two joint hip flexors
D. Normal one joint and two joint hip flexors

A

Correct Answer: B
The posterior thigh does not touch the table, and the knee can be flexed as many degrees beyond 80 degrees as the hip is flexed. The Thomas test is utilized to test for hip flexor length and to distinguish between one joint and two joint hip flexor tightness. With low back and sacrum flat on the table, a normal one joint hip flexor length would be with thigh flat on the table. Normal tow joint hip flexor length would be 80 degrees of knee flexion.

96
Q

A therapist is instructing the family of a 9-year-old boy with Duchenne’s muscular dystrophy (MD). What should be the main focus of the plan of care for maintaining function in the lower extremities?
A. Strengthening the knee extensors and plantar flexors
B. Strengthening the plantar flexors and stretching the hip extensors
C. Stretching the hip flexors and plantar flexors
D. Strengthening the hip flexors and knee extensors

A

Correct Answer: C
Duchenne’ MD is a rapidly progressive disorder characterized by muscle wasting and atrophy.
Contractures of the hips, knees. plantar flexors, and iliotibial band are common. Scoliosis occurs at around age 11 or 12. The main focus is preventing contractures, maintaining activities of daily living (ADL), energy conservation, family education, and positioning.

97
Q

A soccer player with a Q angle in excess of 30 degrees exhibits abnormal patellofemoral tracking. While playing soccer, what is the MOST often used device to address this problem?
A. Patellar stabilizing brace with a lateral buttress
B. Patellar stabilizing brace with a medial buttress
C. Neoprene sleeve with a patellar cutout
D. Derotation brace

A

Correct Answer: A
An increased valgus deformit can result in a greater lateral disnlacement force on the patella, which can disrupt patella tracking and could even lead to subluxation. The theory behind the lateral buttress brace is that it provides support to help prevent subluxation and tries to maintain the normal patella tracking.

98
Q

A patient with diabetes mellitus has had a stage III decubitus ulcer over the right ischial tuberosity for the past 5 months. The ulcer is infected with staphylococcus aureus, and necrotic tissue covers much of the wound. What therapeutic modality is CONTRAINDICATED in this situation?
A. Low-voltage, constant microamperage direct current
B. High-voltage monophasic pulsed current
C. Alternating/biphasic current
D. Moist hot packs

A

Correct Answer: D
Both a moist environment and heat can accelerate bacterial growth. Hot packs would be contraindicated in this case.

99
Q

A therapist is treating a child with spastic diplegia. What intervention can be used to promote relaxation?
A. Rhythmic stabilization
B. Slow rocking on a therapy ball
C. Spinning in a hammock
D. Rolling and spinning on a scooter board

A

Correct Answer: B
Relaxation can be achieved using slow rocking (slow vestibular stimulation).

100
Q

Capsular tightness has limited a patient’s ability to fully extend the left knee. What joint mobilization technique should be used to restore joint motion?
A. Anterior glide and external rotation of the tibia
B. Anterior glide and internal rotation of the tibia
C. Posterior glide and external rotation of the tibia
D. Posterior glide and internal rotation of the tibia

A

Correct Answer: A
The concave-convex rule dictates roll and slide in the same direction. Joint mobilization to increase knee extension should emphasize anterior tibial glide. Anatomically, the medial femoral condyle is longer (has more articular surface) than the lateral; therefore, for the tibia to maintain maximum contact during the last 30 degrees (most evident during the last 5 degrees) of knee extension, external rotation of the tibia on the femur occurs in an open-chain movement.
This happens because the shorter lateral tibial plateau/condyle pair completes its rolling and sliding motion before the longer medial articular surface does. The continued anterior motion of the medial tibial condyle results in external rotation of the tibia on the femur.

101
Q

A group of 37 adolescent girls ages 12 and 13 were recruited into a study on developmental changes in bone density. Bone mineral density (BMD) was measured on admission into the study and every 4 months thereafter for a period of 3 years. The investigators concluded that BMD peaked at the age of 14, with a standard deviation of 7 months. What is the category of this research?
A. Randomized controlled trial
B. Case control study
C. Between-subject study
D. Prospective cohort study

A

Correct Answer: D
This is a prospective (forward-in-time) stud. A group of participants (cohort) with a similar condition is followed for a defined period of time. This is a cohort study.

102
Q

The results of isokinetic dynamometry at 180 degrees per second indicate that a male patient can generate a peak torque of 120 ft Ibs with the right quadriceps muscle and only 80 ft Ibs with the right hamstring musculature. What situation is being described?
A. Quadriceps torque generation is excessive
B. Hamstring torque generation is insufficient
C. Both quadriceps and hamstring torque generation is excessive
D. Torque generation is proportionally correct

A

Correct Answer: D
Hamstring/auadriceps torque generation at various speeds in 15-to-45-year-old males:
* 65% at 60 degrees/sec
* 69% at 180 degrees/sec
* 71% at 300 degrees/sec

103
Q

When using continuous ultrasound in treating the hip of an obese patient, the GREATEST benefit might occur if the ultrasound frequency and dosage are set at which parameters’?
A. 1 MHz and 1.5 watts/cm?
B. 1 MHz and 0.5 watts/cm?
C. 3 MHz and 1.5 watts/cm?
D. 3 MHz and 0.5 watts/cm?

A

Correct Answer: A
1 MHz requency is recommended for target tissue deeper than 2 cm, and 1.5 watts/cm2 would increase the rate of heating, allowing it to be treated in a reasonable time frame.

104
Q

A patient presents with a stage III pressure ulcer with a moist, necrotic wound. A hydrocolloidal dressing (DuoDERM) is being used. During the dressing change, the therapist detects a strong odor, and the wound drainage has a yellow color. What is the therapist’s BEST course of action?
A. Reapply a new gauze dressing instead of hydrocolloid and report the findings to the physician
B. Speak to the nurse about changing to a hydrogel dressing
C. Leave the dressing off the wound and report the findings immediately to the physician
D. Reapply a new DuoDERM dressing and record the findings in the chart

A

Correct Answer: D
Hydrocolloidal dressings are typically changed every 3 to 5 days or when drainage leaks out.
An odor and yellowish color is to be expected as the dressing material melts.

105
Q

During examination of a patient with degenerative osteoarthritic changes in the carpometacarpal (CMC) joint of the right thumb, the physical therapist notes a 20-degree loss of thumb palmar abduction. What translator joint play motion (based on the traditional concave/convex rules of motion) is associate with thumb palmar abduction and should be examined?
A. Dorsal translation of the metacarpal on the trapezium
B. Palmar translation of the metacarpal on the trapezium
C. Ulnar translation of the metacarpal on the trapezium
D. Radial translation of the metacarpal on the trapezium

A

Correct Answer: A
The carpometacarpal joint of the thumb is considered a saddle joint in which the articular surface geometry is generally concave in one plane and convex in a plane perpendicular to the other. The proximal joint surface of the first metacarpal is generally convex in the palmar to dorsal direction and concave in the medial to lateral direction. The articular surface of the base of the first metacarpal typically presents as the convex member of this joint when movement occurs in palmar abduction. Thumb palmar abduction thus involves a convex metacarpal surface moving on the concave surface of the trapezium. Following the traditional concave/convex rules of motion, one would expect a combination of palmar roll and dorsal translator motion of the metacarpal on the trapezium during palmar abduction. In this case, a therapist would be sure to evaluate dorsal glide of the metacarpal on the trapezium.

106
Q

Following a cerebrovascular accident involving the right hemisphere, a male patient is exhibiting unilateral neglect. What might he do as a result?
A. Eat food only from the left side of a plate
B. Bump his wheelchair into things on the right side
C. Ignore or deny the existence of the right upper extremity
D. Shave only on the right side of the face

A

Correct Answer: D
A patient with a right hemisphere lesion (left hemiplegia) will tend to ignore items or body parts on the left side while favoring items or body parts on the right side.

107
Q

Damage as a result of Salter-Harris type IV supracondylar humeral epiphyseal fracture in a young athlete will MOST LIKELY result in what consequence?
A. Refracture at a future time
B. Nonunion
C. Arrested growth
D. Severing of the radial nerve

A

Correct Answer: C
Supracondylar fractures are the most common pediatric elbow fracture, occurring most commonly between 3 and 10 years of age. Extension fractures account for about 95% of supracondylar fractures. The mechanism of injury is a fall on an outstretched hand with elbow hyperextended. Type Is a nondisplaced fracture across the growth plate. Type iNsupracondylar fractures are angulated and displaced fractures across the growth plate and continuing up through the shaft of the bone(Type IN fractures may be displaced in three directions: posteromedial (the most common pattern), posterolateral, or anterolateral. A type III fracture also starts through the growth plate but turns and exits through the end of the bone and into the adjacent joint. Type I is a fracture through all three elements of the bone: the growth plate, metaphysis, and epiphysis (10% incidence). Type IV growth plate fracture starts above the growth plate, cross the growth plate, and exit through the joint cartilage. These injuries can affect the joint cartilage and may impair normal growth. Complete growth retardation or partial growth arrest may result in progressive limb-length discrepancies. Complete growth arrest is uncommon and depends on when the injury to the physis occurs in relation to the remaining skeletal growth potential. The younger the patient, the greater is the potential for problems associated with growth. Premature partial growth arrest is far more common and can appear as peripheral or central closures.

108
Q

What is the BEST way to monitor the intensity of exercise for a patient limited mostly by claudication?
A. Assessing ankle-brachial index (ABI) during exercise
B. Maintaining heart rate (HR) between 60% and 70% of age-predicted HRmax during exercise
C. Sustaining pain levels of at least 2 out of 4 on the claudication scale during exercise
D. Upholding rate of perceived exertion (RPE) levels of 11 to 13 out of 20 during exercise.

A

Correct Answer: C
It has been established that in order to generate collateral circulation in patients with ischemia (i.e., claudication), patients need to exercise with at least moderate claudication pain. This level of blood and oxygen deprivation over time initiates the generation of collateral circulation. This correlates to 2 out of 4 on the claudication scale.

109
Q

A weightlifter with hypertrophy of the scalene muscles complains of pain and paresthesia in the right upper extremity when lifting weight overhead. What is the MOST LIKELY cause?
A. Thoracic outlet syndrome
B. Vertebral artery obstruction
C. Cervical radiculopathy
D. Complex regional pain syndrome type 1

A

Correct Answer: A
Hypertrophied scalene muscles can result in thoracic outlet syndrome due to their close anatomical relationship to the neurovascular structures. The neurovascular bundle passes between the anterior and middle scalene muscles and could be under pressure from hypertrophied scalenes. The anterior and middle scalenes attach to the first rib, and tightness in these muscles could result in elevation of the first rib, thereby compressing the neurovascular bundle. Neurogenic (neurological) thoracic outlet syndrome is characterized by compression of the brachial plexus. In the majority of thoracic outlet syndrome cases, the symptoms are neurogenic. Signs and symptoms of neurological thoracic outlet syndrome often include wasting in the thenar area, numbness or tingling in the fingers, pain in the shoulder and neck, ache in the arm or hand, and weakening grip.

110
Q

A patient is immersed up to the neck in a therapeutic pool. While exercising this patient, the therapist should take into consideration the physiological effect of immersion. Which significant result might occur?
A. Increased forced vital capacity
B. Increased expiratory reserve volume
C. Increased work of breathing
D. Decreased pulmonary blood flow

A

Correct Answer: C
Full chest immersion in a pool can result in increased work of breathing as a result of increased hdrostatic pressure.

111
Q

What intervention BEST illustrates selective stretching when working with a patient with a spinal cord injury (C6 complete?
A. Longer finger flexors are fully ranged into extension with wrist extension
B. Hamstring are fully ranged to 110 degrees in supine
C. Low back extensors are fully ranged in longsitting
D. Hamstrings are full ranged in longsitting

A

Correct Answer: B
Hamstrings need to be fully ranged to 110 degrees in the supine position. This allows for function in the longsitting position (e.g., dressing, leg management during transfers).

112
Q

Strengthening of the lateral pterygoid, anterior head of the digastric muscle, and suprahyoid muscles would be the MOST BENEFICIAL intervention to improve which of the following?
A. Mouth closing
B. Mouth opening
C. Mouth protrusion
D. Mouth retrusion

A

Correct Answer: B
The muscles involved in opening include the lateral ptergoid, anterior head of the digastric muscle, and suprahyoid muscles.

113
Q

A physical therapist examined the deep tendon reflexes of a patient recently diagnosed with amyotrophic lateral sclerosis (ALS). The results are 2+ left and 4+ richt. What is the correct interpretation of these findings regarding the reflexes?
A. Abnormal on the left and normal on the right
B. Normal on the left and brisker, possibly abnormal on the right
C. Depressed, low normal on the left and normal on the right
D. Normal on the left and abnormal on the right

A

Correct Answer: D
Reflexes are average, normal (2+) on the left and very brisk, hyperactive, and abnormal (4+) on the right.

114
Q

What is the BEST evidence to determine orthotic intervention to prevent inversion ankle sprains?
A. Meta-analyses of cohort studies
B. Systematic reviews of randomized controlled trials
C. Meta-analyses of multiple case studies
D. Randomized double-blind controlled trials

A

Correct Answer: B
Systematic review including meta-analysis of randomized controlled trials (RCTs) provides the best research evidence of effectiveness of an intervention.

115
Q

A patient presents with insidious onset of low back pain that started 10 days ago.
Examination reveals an Oswestry Disability Index score of 40%, a Fear Avoidance Belief Questionnaire for Physical Activity score of 16,) hypomobility at L4-5 with posteroanterior (PA) glide, and pain that radiates into the right buttock. Lumbar active range of motion (AROM) is painful at 50% of expected range in all directions. Hip passive range of motion (PROM) is within normal limits. Abdominal strength is fair. Based on this data, what is the BEST intervention for this patient?
A. Abdominal stabilization
B. Manipulation
C. Positional distraction
D. Transcutaneous electrical nerve stimulation (TENS) and ice

A

Correct Answer: B
If a patient presents with four out of five criteria listed in the examination findings, the patient has a 95% chance of benefiting from manipulation using the lumbopelvic regional thrust manipulation technique.

116
Q

Severe epigastric and abdominal pain that radiates to the middle back and may worsen when lying supine is MOST characteristic of which condition?
A. Small intestine obstruction
B. Irritable bowel syndrome
C. Appendicitis
D. Acute pancreatitis

A

Correct Answer: D
Pancreatitis occurs midline or to the left of the epigastrium, just below the xiphoid process. Pain is referred to the middle or lower back and rarely to the upper back.

117
Q

A physical therapist is working with a patient recovering from traumatic brain injury (Ranco Los Amigos Levels of Cognitive Functioning Scale level VII). The best test to determine if this patient demonstrates progression to open skills is to have the patient walk in which environment?
A. In the hallway
B. Across the busy hospital lobby
C. In the patient’s room
D. In the patient’s home

A

Correct Answer: B
Open skills are motor skills (e.g., walking) that can be performed in a variable, changing environment (a busy hospital lobby).

118
Q

A patient with a 7-year history of Parkinson’s disease is hospitalized. The patient is ambulatory but requires close supervision to prevent falls. What should be the focus of the physical therapist’s plan of care?
A. Manual balance perturbation training
B. Transfer and wheelchair training
C. Caregiver training for contact guarding during level walking and stairs
D. Locomotor training using a rolling walker

A

Correct Answer: C
Caregiver training with safety instruction in contact guarding during level walking and stairs is the best choice to keep this patient functional in the home environment.

119
Q

A patient is referred to physical therapy with a diagnosis of congestive heart failure.
During the initial session, the physical therapist examines the skin for suspected changes.
What appearance can be expected?
A. Pale, washed-out color
B. Yellowish discoloration
C. Slightly bluish, slate-colored discoloration
D. Cherry-red discoloration

A

Correct Answer: C
Slightly bluish, grayish, slate-colored discoloration of the skin along with clubbing of the nails is characteristic of chronic hypoxia.

120
Q

A patient with a 10-year history of discoid lupus ervthematosus presents with multiple discoid skin lesions that are raised and red and contain scaling plaques with central atrophy on the lower extremities. Topical corticosteroid creams are being used, What should be the focus of the therapist’s initial plan of care?
A. Range of motion (ROM) exercises and prevention of deformity
B. Lightweight splints to provide joint protection
C. Aerobic training using a treadmill
D. Resistive training using weights at 60% to 80%, one-repetition maximum

A

Correct Answer: A
Range of motion (ROM exercises and prevention of deformity are important elements of the plan of care.

121
Q

On the third day following a cesarean delivery, what should a physical therapist’s interventions include?
A. Gentle partial sit-ups and head lifts
B. Breathing, coughing, and pelvic floor exercises
C. Low-intensity aerobic conditioning
D. Pelvic tilts on all fours

A

Correct Answer: B
Initial postpartum interventions (days 1 to 3) should include breathing, coughing, and pelvic
floor exercises.

122
Q

A patient with post-traumatic brain iniury (Rancho Los Amigos Levels of Cognitive Functioning Scale level III) has evidence of retained secretions on auscultation and chest films. What is the BEST mode of airway clearance for this patient?
A. Active cycle of breathing
B. Autogenic drainage
C. Use of the FLUTTER device
D. Use of the Vest Airway Clearance System

A

Correct Answer: D
The Vest Airway Clearance System provides high-frequency chest wall compression
(HFCWC). It allos for control of inspiratory and expiratory flow rates. The device can be used in any position regardless of the patient’s cognitive status.

123
Q

A physical therapist is performing sensory tests on a patient diagnosed with C6 nerve root impingement. Where should the testing concentrate?
A. Second, third, and fourth fingers, palmar surface
B. Ulnar border of the hand (fifth finger)
C. Palmar surface of the thumb and distal, radial forearm
D. Medial (ulnar) forearm

A

Correct Answer: C
The C6 root supplies the palmar surface of the thumb and distal radial forearm.

124
Q

A physical therapist is treating an elderly patient in the very early stages of amvotrophic lateral sclerosis (ALS). The patient lacks endurance to independently ambulate to and from the bathroom; however, she refuses to use a bedpan. This has caused added difficulties and stress for family members caring for her at home. What should the therapist do in this situation?
A. Initiate ambulation endurance training with distance to and from the bathroom as the primary goal
B. Recommend that the patient be transferred to a skilled nursing facility until she has sufficient endurance to get to and from the bathroom
C. Recommend use of a commode chair for the bedroom
D. Recommend use of a wheelchair and toilet grab rails

A

Correct Answer: C
The bedside commode is the best choice to improve function.

125
Q

If the sign indicated by the picture is positive, intervention should focus on strengthening which of the following?
A. Left gluteus medius
B. Right gluteus medius
C. Left quadratus lumborum
D. Right quadratus lumborum

A

Correct Answer: A
This picture depicts a positive Trendelenburg sign. Weakness of the left hip abductors (gluteus medius) results in dropping of the pelvis on the opposite side. A normal response would be no dropping of the pelvis during the standing test.

126
Q

A physical therapist is working with a patient who exhibits fluent aphasia. What is a typical characteristic of this form of aphasia?
A. Impaired auditory comprehension
B. Slow, hesitant speech
C. Good comprehension
D. Impaired articulation

A

Correct Answer: A
Fluent aphasia is characterized by impaired auditory comprehension and fluent speech that is of normal rate and melody (e.g., Wernicke’s aphasia).

127
Q

Three months following a left cerebrovascular stroke and a 4-week stay of inpatient rehabilitation, a patient is receiving home care physical therapy. The patient’s movements in the right extremities show good recovery (out-of-synergy). Functional level is a 6 on the Functional Independence Measure (FIM). At this juncture, what should be the focus of motor learning strategies?
A. Use of mental practice to improve performance
B. Breaking down complex tasks into component parts
C. Use of serial practice order of related skills
D. Consistency of performance in variable environments

A

Correct Answer: D
This patient demonstrates good functional recovery. Motor learning for the autonomous stage of motor learning should utilize variable practice in variable environments.

128
Q

When performing neutral tension testing to the lower limb, which ankle position when combined with a straight leg raise will BEST bias the peroneal (fibular) nerve?
A. Dorsiflexion and eversion
B. Dorsiflexion and inversion
C. Plantarflexion and eversion
D. Plantarflexion and inversion

A

Correct Answer: D
Neural tension tests are used to determine neural mobility in relation to its surrounding tissue. It should be noted that patients will experience symptoms even when normal neural tissue is tensioned. It is paramount for the examiner to understand the expected normal responses with tension testing. A tension test can be considered positive if it reproduces the patient’s symptoms, the test response can be altered by movement of distant body parts, and there are differences when comparing the left side to the right side.
Neural mobilization techniques including gliders/sliders and tensioners are interventions used to decrease adverse mechanical tension on nerves. A straight leg raise with plantarflexion and inversion is the optimal position for neural tissue provocation of the peroneal (fibular) nerve.

129
Q

What is the most effective form of diagnostic imaging for patients with multiple sclerosis
(MS) to help determine level of disease activity?
A. Position emission tomography (PET)
B. Magnetic resonance imaging (MRI)
C. Computed tomography (CT)
D. Transcranial sonography

A

Correct Answer: B
MRI is highly sensitive for detecting MS plaques in the white matter of the brain and spinal cord. Cesions are seen as areas of increased signal intensity (bright spots). Contrast-enhanced scans are used for more long term disease activity.

130
Q

An elderly male patient is not able to participate in rehabilitation. He is lethargic, complains of nausea and painful urination, and seems to be feverish. The therapist should inform his primary care physician if which of the following is suspected?
A. Bladder cancer
B. Benign prostatic hyperplasia
C. Urinary tract infection
D. Renal calculi

A

Correct Answer: C
These are signs and symptoms of urinary tract infection. Evidence of fever is especially significant. The physician should be informed.

131
Q

The left phrenic nerve of a patient was accidentally severed during thoracic surgery.
Which muscles should the physical therapist strengthen in order to provide substitute function?
A. Transversus abdominis
B. Scalenes
C. Internal obliques
D. External obliques

A

Correct Answer: B
The phrenic nerve arises from the neck (C3-5) and innervates the diaphragm. The diaphragm is responsible for 45% of the air that enters the lungs during quiet breathing. During quiet breathing, the predominant muscle of respiration is the diaphragm. As it contracts, pleural pressure drops, which lowers the alveolar pressure and draws in air down the pressure gradient from mouth to alveoli. Expiration during quiet breathing is predominantly a passive phenomenon; as the respiratory muscles relax, the elastic lung and chest wall return passively to their resting volume. With paralvsis of the diaphragm, the accessory muscles of respiration should be strengthened. These include the scalenes and sternocleidomastoid.

132
Q

A patient recovering from surgery to remove a cerebellar tumor presents with pronounced ataxia and problems with standing balance and postural stability. To help improve this situation, what would be BEST approach to incorporate in the intervention?
A. Lower extremity splinting and light touch-down hand support
B. Rhythmic stabilization during holding in kneeling
C. Perturbed balance activities while standing on carpet
D. Stabilizing reversals during holding in side-lying

A

Correct Answer: B
Rhythmic stabilization is proprioceptive neuromuscular facilitation (PNF) technique designed to improve stability. The high kneeling position is a good choice to begin with for the patient with pronounced ataxia. The posture is upright; while the center of mass (COM) is lowered, the degrees of freedom are reduced by kneeling (foot and ankle control not required and the base of support (BOS) is increased over standing.

133
Q

A patient with diabetes recently underwent a transtibial amputation. When instructing others in wrapping the residual limb to minimize edema and provide proper shape, it is important to stress which of the following?
A. The greatest pressure should be provided distally
B. The greatest pressure should be provided proximally
C. The pressure should be evenly distributed
D. An overlapping, circular, wrinkle-free pattern should be used

A

Correct Answer: A
It is necessary to keep the residual limb wrapped to help shape it for possible prosthetic wear and to control swelling. It should be wrapped at all times except when bathing. Elastic wraps should be used initially. Wrap with the knee in as much extension as possible. Use two clean, rolled, bandages (4-6 inches wide). Use wraps that close with Velcro if possible or use tape to fasten rather than clips. Rewrap if the bandage is uncomfortable or if it slips. Make all turns diagonal, not circular. Circular turns could decrease circulation and produce discomfort. Make sure that the bandage is smooth, without wrinkles, and that all of the skin of the stump is covered and overlapped by ½ inch with each wrap. The pressure should be more at the bottom of the limb to help form the limb into a cylinder shape. To help prevent the wrap from slipping, continue the wrap above the knee one or two turns but finish the wrap on the residual limb.

134
Q

A patient has worsening lateral knee pain that is exacerbated with running. Based on the history and symptoms, the therapist suspects iliotibial band friction syndrome. Which special test is performed that would help confirm this diagnosis?
A. Stutter test
B. Thessaly test
C. Ely’s test
D. Noble compression test

A

Correct Answer: D
The Noble compression test implicates iliotibial band friction syndrome. This test is performed with the patient supine. The examiner flexes the patients’ knee and hip to 90 degrees and then applies pressure to the lateral femoral epicondyle with the thumb. While maintaining this position, the knee is passively extended. A positive test is reproduction of pain at approximately 30 degrees of knee flexion.

135
Q

Six weeks following the conclusion of the football season, a therapist examines a player whose chief complaint is right thigh pain and decreased knee range of motion. Radiographic imaging of the area is shown in the picture. Intervention for this individual should be based on which diagnosis?
A. Femoral stress fracture
B. Neoplasm
C. Quadriceps hematoma
D. Myositis ossificans

A

Correct Answer: D
Soft tissues that were injured in a traumatic event initially develop a hematoma and subsequently can develop into myositis ossificans. Mvositis ossificans is a benign, ossifying soft-tissue lesion typically occurring within skeletal muscle. Patients are usually adolescents and young adults. Myositis ossificans is rare in children under 10 years of age. The most frequent symptoms and signs are pain and tenderness with a soft tissue mass. Approximately 80% of cases arise in the large muscles of the extremities.

136
Q

During a finger-to-nose test, a patient demonstrates hesitancy in getting started and is then unable to control the movement. The finger slams into the side of the face, missing the nose completely. How should the therapist document this finding?
A. Dysmetria
B. Dysdiadochokinesia
C. Dyssynergia
D. Intention tremor

A

Correct Answer: A
Dysmetrid is an inability to judge the distance or range of movement. It includes both overestimation (hypermetria) and underestimation (hypometria) of the required range needed to reach the goal.

137
Q

What is the primary reason a patient with advanced chronic obstructive pulmonary disease (COPD) has improved/increased aerobic capacity with bilateral upper extremity support during activity training?
A. It facilitates reverse action latissimus dorsi function to maximize ventilation
B. Holding on increases the resistance or workload
C. It increases sensory input and decreases balance work
D. It improves scalene recruitment to improve expiration

A

Correct Answer: A
Stabilizing the unver extremities allows the latissimus dorsi to function such that it lifts the rib cage, thereby increasing negative pressure within the thorax and improving ventilation.

138
Q

A physician requests that a physical therapist perform hydrocortisone iontophoresis over the left shoulder of a patient with tendonitis. The therapist discovers that the patient has a pacemaker. In this case, what should the therapist do?
A. Perform the treatment since there is no contraindication
B. Refer the patient to another physical therapist who has greater expertise in using iontophoresis for patients with pacemakers
C. Consult with the physician about alternative forms of therapy; do not perform the 10ntophoresis treatment
D. Administer the iontophoresis but only on the top of the shoulder not near the chest wall

A

Correct Answer: C
All applications of electrical stimulation are contraindicated in the presence of a pacemaker.
Consultation with the referring physician is necessary.

139
Q

Normotensive overweight and obese adolescents are performing isometrics and weightlifting as part of an exercise program. What is an important factor for the physical therapist to consider?
A. This type of exercise will decrease lean body mass and assist in weight loss
B. Blood pressure will fall if participants continuously hold their breath at the beginning of these exercises
C. This type of exercise will not affect cardiovascular conditioning
D. Blood pressure will rise during exercise and fall below resting levels after exercise ceases

A

Correct Answer: D
Blood pressure can be expected to rise during isometrics and weightlifting and fall below resting levels after exercise ceases. This is a normal hemodvnamic response to exercise.

140
Q

During gait analvsis, a therapist notes that a patient is lurching backward during stance phase. What is the cause of this compensatory motion?
A. Gluteus medius weakness
B. Hip and knee flexion contractures
C. Quadriceps weakness
D. Gluteus maximus weakness

A

Correct Answer: D
Lurching backward during stance is a compensation for weak hip extensors, commonly called a gluteus maximus gait. Leaning backward during loading response inclines the ground reaction force vector posteriorly from point of application at the hindfoot. Because the vector passes closer to the hip joint’s lateral axis, its moment arm is shorter, and it produces a smaller hip flexors moment. If it falls behind the axis it can produce a hip extensor moment.

141
Q

What are some common adverse effects that patients taking nitrates, diuretics, beta-blockers, or calcium antagonists might experience?
A. Hypotension and dizziness
B. Arrhythmia and unstable blood pressure
C. Extreme fatigue and arrhythmias
D. Hypotension and decreased electrolytes

A

Correct Answer: A
All of these medications lower blood pressure. If the dosage is too great for patients, they will be hypotensive and likely feel dizzy.

142
Q

A 3-year-old child with Arnold-Chiari malformation has a ventriculoperitoneal shunt in place. During physical therapy treatment, the child becomes agitated and irritable, then drowsy and listless. What should the therapist do in this situation?
A. Immediately place firm pressure over the fontanel
B. Administer emergency oxygen
C. Place the child in a head-down position
D. Call for emergency medical services

A

Correct Answer: D
These are all signs of shunt blockage Emergency medical services are indicated.

143
Q

A patient with a 6-year history of Parkinson’s disease (PD) has experienced two recent bouts of pneumonia and limited functional mobility in the home. The therapist’s plan of care focuses on improving respiratory function and postural control. What is the BEST choice for intervention to address these issues at this time?
A. Supine, UE PNF lift and reverse lift patterns using rhythmic initiation
B. Quadruped, alternate arm and leg raises
C. Sitting, bilateral symmetrical UE PNF D2 flexion patterns using rhythmic initiation
D. Standing, bilateral symmetrical UE PNF D2 flexion patterns using dynamic reversals

A

Correct Answer: C
Sitting, bilateral symmetrical UE PNF D2 flexion patterns using rhythmic initiation are the best choices to open up the chest and enhance lung function (restrictive lung function is common in patients with PD). The sitting posture is a good starting position for a patient with postural instability since the base of support (BOS) is wide and the center of mass (COM) is lowered compared to standing.

144
Q

An elderly patient with diabetic peripheral neuropathy and retinopathy is having difficulty with balance when ambulating at home. The patient has fallen three times in the last month. What is the first priority of the home physical therapist’s plan of care?
A. Gait training with a cane to ensure safety
B. Color-coding raised surfaces, such as steps, with a sharp color contrast
C. Ambulation practice on changing floor to carpet surfaces in the home
D. Installing nightlights in strategic areas throughout the house and keeping them lit continuously

A

Correct Answer: A
The first priority of the home physical therapist should be gait training with a cane to ensure safety. This compensatory strategy is necessary as this patient is demonstrating complications of diabetes, which are chronic and progressive.

145
Q

What is the minimal recommended work interval duration for a nondeconditioned adult performing aerobic interval training?
A. 5 minutes
B. 10 minutes
C. 20 minutes
D. 30 minutes

A

Correct Answer: B
Guideline for interval training call for performance of intermittent exercise of at least 10 minutes of duration (American College of Sports Medicine).

146
Q

A therapist is monitoring the blood pressure of a healthy athlete exercising on a treadmill.
The speed and incline steadily increase during the exercise period. The therapist would expect the blood pressure response to demonstrate which of the following?
A. Blunted rise in systolic pressure and a slight decrease in diastolic pressure
B. Slight drop in systolic pressure and either a slight increase or decrease in diastolic
pressure
C. Steady increase in systolic pressure accompanied by a steady increase in diastolic
pressure
D. Steady increase in systolic pressure and either a slight increase or decrease in diastolic pressure

A

Correct Answer: D
A steady increase in systolic pressure and either a slight increase or decrease in diastolic pressure is a normal response to ramp exercise protocol. With a continual, steady increase in exercise, the systolic blood pressure will continue to rise because the patient si not permitted to reach steady state.

147
Q

A therapist is examining a patient with a temporomandibular disorder. The patient had a history of jaw clicking upon opening and closing the mouth, but the noises are now gone.
The current major complaint is inability to open the mouth wide and difficulty performing functional jaw movements such as chewing and yawning. What is the most likely cause of this current impairment?
A. Capsular fibrosis
B. Temporomandibular joint (TMJ) subluxation
C. Chronic TMJ osteoarthritis
D. Disc displacement without reduction

A

Correct Answer: D
The mandible can open partway without clicking because the condyle is able to push the dislocated disc in front of it until the ligaments become taut. Further opening causes the chin to deviate off to the affected side because the affected condyle stops moving forward while the condyle of the other side keeps moving forward. Further jaw opening requires the affected side to jump over the back end of the dislocated disc and onto its thin center, causing a click or pop.
Thus the opening click occurs near wide opening because it doesn’t occur until the ligaments attached to the backs of the disc become taut; and the closing click is softer and occurs when the jaw is more nearly closed because the disc doesn’t re-dislocate until the condyle reaches the depth of the socket where the disc can no longer fit. In later stages of forward disc dislocation, the disc usually stops going back into place during mouth opening, and the mandible cannot open past the point where it used to click.

148
Q

A patient recovering from a total hip arthroplasty is seen by the physical therapist for early mobilization out of bed. While sitting on the edge of the bed, the patient experiences rapid onset of dyspnea, sudden chest pain, and cyanosis. What action should the therapist take?
A. Return the patient to supine and monitor vital signs for the next 5 minutes
B. Stabilize the patient and contact medical services immediately
C. Allow the patient to rest for a few minutes and continue with the therapy session
D. Return the patient to supine and reschedule the therapy session for later in the afternoon

A

Correct Answer: B
This patient is exhibiting signs and symptoms of pulmonary embolism. This is an emergency
medical situation and a cause of death in a substantial number of patients.

149
Q

A patient with traumatic brain inury exhibits strong spasticity in both lower extremities (limbs are held rigid in extension). Using the Modified Ashworth Scale, the therapist gives this response which of the following grades?
A. 1+
B. 2
C. 3
D. 4

A

Correct Answer: D
A grade of 4 is given for affected parts) that are held rigid in flexion or extension.

150
Q

When working with a child with Down syndrome and severe hypotonicity, how would it be best to activate the postural extensor muscles during early intervention?
A. Slow, repetitive rocking movements with the child seated on a large gymnastic ball
B. Prone positioning on a large gymnastic ball with the child looking up
C. Quadruped, opposite arm and leg lifts
D. Standing, weight shifts in modified plantigrade.

A

Correct Answer: B
The child with Down syndrome typically demonstrate hypotonia, developmental delay in postural stability, and poor muse of propriocention for postural control. Prone positioning on a large gymnastic ball and having the child look up (neck extension) and/or reach up is a good early intervention to activate the postural extensors.

151
Q

A patient recovering from stroke demonstrates dyspraxia. On what should physical therapy intervention optimally focus?
A. Reeducation of weak muscles using isokinetics before activity practice
B. Compensatory training strategies with maximum use of environmental cues
C. Task-specific practice of familiar activities progressing from parts to whole
D. Maximum use of manual facilitation of movements and new tasks

A

Correct Answer: C
Dyspraxia is an impairment of skilled learned movement (a disconnect between the idea for movement and its motor execution). Task-specific practice using familiar activities and progression from parts to whole is the best choice to enhance learning.

152
Q

A patient presents with a limitation of wrist flexion. The joint manipulation technique that would BEST improve the patient’s range of motion is which of the following Grade IV descriptions?
A. Posterior to anterior glide of the proximal carpal row on distal radius and ulna
B. Posterior to anterior glide of the lunate on capitates
C. Anterior to posterior glide of the proximal carpal row on distal radius and ulna
D. Anterior to posterior glide of lunate on capitates

A

Correct Answer: C
A Grade IV anterior to posterior glide of proximal carpal row on distal radius and ulnar is the correct choice because during wrist flexion, the proximal row moves dorsally on the distal radius and ulna.

153
Q

A therapist wishes to determine the fall risk of an elderly person in a long-term care facility. Which test of static balance would be most difficult for this person to perform?
A. Standing unsupported with feet together, eyes open
B. Standing unsupported with one foot in front, tandem position, eyes closed
C. Standing unsupported with eyes closed
D. Standing unsupported on one leg, eyes open

A

Correct Answer: B
These are all measure of static standing balance found on the Berg Balance Scale (BBS). The most difficult test is tandem standing (reduced based of support [BOS] with eyes closed (EC).

154
Q

A patient with insulin-dependent diabetes is participating in an aerobic exercise class.
The therapist recognized that important dietary recommendations to prevent delaved hypoglvcemia after exercise include intake of which of the following?
A. Fruit juice or candy
B. Crackers or bread
C. Steak or chicken
D. Salad or cooked vegetables

A

Correct Answer: B
Slowly absorbed carbohydrates (crackers, bread, or pasta) can help prevent delaved-onset hypoglycemia.

155
Q

A physical therapist is examining a patient who is complaining of pain in the left shoulder region. The examination of the shoulder elicits pain in the last 30 degrees of shoulder abduction range of motion. This finding is most congruent with which diagnosis?
A. Calcific supraspinatus tendinitis
B. Subacromial bursitis
C. Acromioclavicular (AC) sprain
D. Thoracic outlet syndrome

A

Correct Answer: C
Typically, AC sprains will have pain at extremes of active range of motion (AROM), especially horizontal adduction and full elevation and pain on passive horizontal adduction and elevation.
There are three special tests for AC joint:
* Acromioclavicular shear test: positive if abnormal movement of AC joint or pain at joint
* Passive cross-chest adduction
* O’Brien test

156
Q

Early intervention programs are usually required to use a standardized and comprehensive developmental test. What is the BEST test for use by a physical therapist?
A. Functional Independence Measure for Children (WeeFIM)
B. Denver Developmental Screening Test (Denver II)
C. Peabody Developmental Motor Scales
D. Movement Assessment of Infants (MAI)

A

Correct Answer: C
The Peabody Developmental Motor Scales is a norm-referenced, standardized assessment of gross motor and fine motor skill divided into six subtests with an age range of 1 to 72 months.

157
Q

A patient presents with an acquired flatfoot deformity. The therapist recognizes that this can result from injury to muscle tendon. Which structure should be examined?
A. Anterior tibialis tendon
B. Posterior tibialis tendon
C. Fibularis longus tendon
D. Achilles tendon

A

Correct Answer: B
The posterior tibial tendon helps hold up the arch up and provides support when stepping off on the toes when walking. If this tendon becomes inflamed, overstretched, or torn, one may experience pain on the inner ankle and gradually lose the inner arch on the bottom of the foot, leading to flatfoot (posterior tibial tendon dysfunction (PTTD]).

158
Q

A 13-year-old girl has a structural right thoracic idiopathic scoliosis. The clinical features a physical therapist would expect to find include which of the following?
A. A high right shoulder, a prominent right scapula, and a left hip that protrudes
B. A high left shoulder, a prominent left scapular, and a right hip that protrudes
C. A high right shoulder, a prominent left scapula, and a right hip that protrudes
D. A high left shoulder, a prominent right scapula, and a left hip that protrudes

A

Correct Answer: A
The Scoliosis Research Society defines scoliosis as a curvature of the spine measuring 10 degrees or greater on x-ray. The condition isn’t rare. It mainly affects girls, many of whom have mild forms of scoliosis, are never even aware of it, and never need treatment. Three to five children out of every 1,000 develop spinal curves that are considered large enough to require treatment. “idiopathic” simply means that there is no known cause. Common signs and symptoms of scoliosis may include:
* Uneven shoulder heights
* Head not centered with the rest of the body
* Uneven hip heights or positions
* Uneven shoulder blade height or positions
* Prominent shoulder blade
* When standing straight, uneven arm lengths
* When bending forward, asymmetrical left and right sides of the back

159
Q

A patient with complete C7 spinal cord iniury is receiving physical therapy in an inpatient rehabilitation setting to maintain joint mobility. What intervention is likely to produce the GREATEST risk of heterotropic ossification (HO)?
A. Prolonged positioning with resting splints
B. Forceful passive range of motion (PROM), especially if spasticity is present
C. Prolonged stretching using tilt table standing
D. Joint mobilization with PROM

A

Correct Answer: B
Forceful PROM in the presence of spasticity increases the risk of developing HQ (osteogenesis typically occurring in the soft tissues adjacent to large joints).

160
Q

A patient complains of vascular changes in the hands usually experienced whenever it is cold. The therapist suspects Raynaud’s disease. Which examination findings are consistent with this diagnosis?
A. Hypersensitivity to tactile stimuli
B. Loss of proprioception of the affected fingers
C. Loss of two-point discrimination in the affected hands
D. Temporary pallor and cyanosis of the digits

A

Correct Answer: D
Raynaud’s disease is a vasospastic disorder characterized by intermittent episodes of small artery constriction of the digits of the fingers (rarely the toes), causing temporary pallor and cyanosis.

161
Q

A patient sustained a trimallolar ankle fracture on the right and a fracture of the left distal radius. For partial weight bearing, it is BEST if the therapist has the patient use which device?
A. Axillary crutches
B. Forearm crutches
C. Platform crutches
D. Lofstrand crutches

A

Correct Answer: C
Platform crutches allow weight bearing on the forearms and are used for patients who are unable to bear weight through their hands, as in this case.

162
Q

Ten weeks after shoulder surgery, a patient develops capsular stiffness that is limiting glenohumeral internal rotation. The physical therapist chooses joint manipulation followed by a supportive exercise to correct this impairment. What is the BEST intervention?
A. Anterior glide of the humerus and functional internal rotation stretch with arm up the back
B. Anterior glide of the humerus and partial push-ups
C. Posterior glide of the humerus and horizontal adduction stretch with arm across chest
D. Posterior glide of the humerus and standing scapular retractions

A

Correct Answer: C
The best intervention is posterior glide of the humerus and horizontal adduction stretch with arm across chest. During glenohumeral internal rotation, there is a posterior glide of the humerus in the glenoid fossa. The best way to self-stretch the posterior capsule is to adduct the arm, putting the posterior capsule and soft tissues on stretch.

163
Q

A teenager is admitted to a skilled nursing facility with a severe traumatic brain injury and marked spasticity. Cognitive function is documented at Rancho Los Amigos Levels of Cognitive Functioning Scale level IV. Family members visit on a daily basis. In this situation, it would be BEST if passive range of motion (PROM exercises are which of the following?
A. Taught to family members in order for them to participate in the care for the patient
B. Performed only by the physical therapist since the patient is unable to follow verbal commands
C. Performed only by the physical therapist (PT) or physical therapist assistant (PTA) to minimize the possibility of pathological fractures
D. Taught to all registered nurses (RNs) who might participate in the care for the patient

A

Correct Answer: A
Passive range of motion (PROM) exercises can be taught to family members in order for them to participate in the care of the patient.

164
Q

After gait training a patient with a transtibial prosthesis, a therapist notices redness along the patellar tendon and medial tibial flare. What would this indicate?
A. The socket is too small and the residual limb is not seated properly
B. The socket is too large and pistoning is occurring
C. There is improper weight distribution during stance
D. Pressure-tolerant weight bearing is occurring

A

Correct Answer: D
Pressure-tolerant areas of the typical transtibial residual limb include the patellar tendon, the medial tibial plateau, the tibial and fibular shafts, and the distal end.

165
Q

During the examination of a 2-vear-old child with mild cerebral palsy, the therapist is encouraged because the normal developmental milestones for a child of this age have been achieved. This was demonstrated by the child’s ability to perform which activity?
A. Hop on one foot
B. Stand on tiptoes
C. Go upstairs foot-over-foot
D. Jump with two feet

A

Correct Answer: C
Going up stairs foot-over-foot (reciprocal stair climbing) is a developmental skill normally achieved by 2 years.

166
Q

A therapist sees a patient in the intensive care unit with multiple trauma and severe traumatic brain injury. A chest tube is in place and it exits from the right thorax. The patient is in need of airway clearance. What action should be taken in this case?
A. Percussion and shaking are contraindicated due to the traumatic brain injury
B. Percussion and shaking can be done only in the right side-lying position
C. Percussion and shaking can be done in the area surrounding the chest tube
D. Percussion and shaking can be done only when the chest tube is removed

A

Correct Answer: C
It is possible to complete manual techniques in the area of the chest tube. It is often the area in most need of airway clearance. It is important to consider pain management when doing this intervention.

167
Q

To prevent maximal compressive forces being placed on the patella, a therapist should minimize placing the patient in which position?
A. Prone and flexing the knee to 30 degrees
B. In a sitting position with the knee flexed to 90 degrees
C. Supine and flexing both the hip and knee to 110 degrees
D. Prone and flexing the knee to 110 degrees with the hip extended

A

Correct Answer: D
Extending the hip while flexing the knee will lengthen the rectus femoris and tensor fascia late, resulting in increased compressive forces at the patellofemoral joint. This position should be avoided if the goal is to minimize this force. At 25 degrees, the force at the patellofemoral joint is equal to that passing through the tibiofemoral joints.

168
Q

To promote upright posture and higher walking speeds in a child with spastic diplegia, which ambulatorv aid is MOST beneficial?
A. A reciprocating gait orthosis
B. An anterior rollator walker
C. A posterior rollator walker
D. A parapodium

A

Correct Answer: C
A posterior rolling walker is used to promote an upright posture (eliminates the forward lean seen in use of the standard walker). The addition of wheels improves walking and reduces energy expenditure.

169
Q

Following an exercise session for patients with heart failure in a Phase 3 cardiac rehabilitation program, what is prevented if the therapist employs a gradual and prolonged cool-down period?
A. Exertional dyspnea
B. Tachycardia
C. Venous pooling
D. Hypertension

A

Correct Answer: C
There is vasodilation while exercising to increase oxygen supply to peripheral tissues. The return to normal arterial dilatation is delaved in patients with heart failure, and a prolonged. gradual cool down is recommended.

170
Q

A therapist wishes to examine the effects of fatigue on physical, cognitive, and functional performance in a patient with a 7-year history of multiple sclerosis (MS). What is the BEST instrument to use?
A. Functional Independence Measure (FIM)
B. Waling ability questionnaire (WAQ)
C. Outcome and Assessment Information Set (OASIS
D. Modified fatigue impact scale (MFIS)

A

Correct Answer: D
The MFIS is an appropriate measure to examine the physical. cognitive and psychosocial functional limitations of fatigue in patients with MS.

171
Q

As a result of diminished movement associated with rigidity in Parkinson’s disease (PD), a physical therapist might employ rhythmic initiation (RI) primarily to help improve which of the following?
A. Trunk stability and proximal tone
B. Trunk rotation
C. Upper extremity function
D. Active and passive range of motion

A

Correct Answer: B
RI is a proprioceptive neuromuscular facilitation (PNF) technique designed to improve mobility. Trunk rotation is particularly lacking in patients with PD and is a good activity to focus on in order to improve bed mobility.

172
Q

A physical therapist wishes to mobilize a patient’s shoulder using an inferior glide technique. It would be BEST to initially use this technique by prepositioning the patient’s arm in which position?
A. 95 degrees of abduction with lateral rotation
B. 125 degrees of abduction and internal rotation
C. 35 degrees of abduction and neutral rotation
D. 95 degrees of shoulder flexion and neutral rotation

A

Correct Answer: C
Joint mobilization generally begins in the loose-pack position but with improvement can progress to more closed-pack positions of the joint. The loose-pack position of the glenohumeral ioint is from 30 to 40 degrees in the scapular plane.

173
Q

What is the Thompson test used to examine?
A. Anterolateral rotational instability of the knee
B. Iliopsoas tightness
C. Rectus femoris tightness
D. Achilles tendon rupture

A

Correct Answer: D
The Thompson test is used to examine the integrity of the Achilles tendon. The patient lies prone with feet off the end of the plinth. The examiner squeezes the calf muscle. This should cause the toes to point downward as the Achilles pulls the foot. The foot will not move in a patient with a totally ruptured Achilles tendon (a positive Thompson test).

174
Q

A physical therapist is treating a terminally ill patient with AIDS at home. What would be a major psychological focus or consideration when managing this patient?
A. Discontinue treatment if the patient/therapist relationship becomes overly dependent
B. Encourage expression of feelings and memories
C. Keep the patient’s friends and relative up to date on the patient’s treatment and state of mind
D. Discontinue any activities that may cause the patient discomfort in order to keep anxiety levels low

A

Correct Answer: B
When treating the patient with a terminal illness, the therapist should provide support and understanding of the grief process, encourage expression of feelings and memories, and respect privacy, cultural, or religious customs.

175
Q

A patient with a transverse spinal cord iniury has total lack of hip flexion, abduction, and knee extension. This functional loss is consistent with a designation of a complete spinal cord lesion at which level?
A. L1
B. L3
C. L4
D. L5

A

Correct Answer: A
Hip flexors are innervated and functional at the L2 (key muscle). Therefore, an L1 lesion would produce complete loss of this muscle function.

176
Q

A physical therapist (PT) is substituting for an ill colleague and is unable to access the previous PT’s notes in the medical record. In this case, what should the therapist do?
A. Ask the patient what treatment had been administered in the last session
B. Attempt to reach the ill therapist by phone before commencing the session
C. See if other coworkers can figure out how to access the information in the medical record
D. Briefly examine the patient and intervene appropriately

A

Correct Answer: D
If a Pt accepts an individual for physical therapy services, the PT will be responsible for the examination, evaluation, and intervention of a patient. At the least, a systems review and brief plan of care should be formulated before intervening. Electronic medical records help to minimize difficulties in handwriting interpretation; however, systems aren’t perfect.

177
Q

A physical therapist receives a referral to examine and treat an elderly patient in the early stage of amyotrophic lateral sclerosis (ALS). A confounding factor is the patient’s mild Alzheimer’s type dementia (AD). A significant factor in patient management at this time will MOST LIKELY be which impairment?
A. Memory impairment
B. Maintaining airway clearance
C. Limited mobility
D. Limited activities of daily living (ADL)

A

Correct Answer: A
The patient’s memory impairment associated with AD will significantly impact management at this time. Significant others or home health aides will be necessary to maintain this patient’s independence in the home.

178
Q

The physical therapist receives a referral to evaluate and treat a 6-month-old infant with right congenital muscular torticollis. On initial examination, the therapist would expect the head to be in which position?
A. Tilted toward the noninvolved side, with the chin rotated toward the same side
B. Tilted toward the involved side, with the chin rotated toward the opposite side
C. Limited in ROM in lateral flexion toward the involved side
D. Limited in ROM in neck flexion and extension

A

Correct Answer: B
Congenital muscular torticollis involves a shortened sternocleidomastoid (SCM) muscle with a weakened contralateral SCM muscle, with a resulting posture of lateral flexion of the head to be involved (right) side, tight SCM muscle side and rotation of the head to the noninvolved (left) side.

179
Q

An elderly female patient is being evaluated for recurrent thoracic back pain. During the history, the patient reveals a smoking habit, and she drinks four to six cups of coffee a day.
Activity level is low, consisting of daily trips to the local coffee shop to socialize with friends. Body weight and height are below normal. Medical history includes Graves disease. The therapist decides to consult the primary physician for further workup. What is the suspected problem?
A. Osteoporosis
B. Osteoarthritis
C. Gout
D. Spinal stenosis

A

Correct Answer: A
This patient is exhibiting several risk factors for osteoporosis: post-menopausal age, low body weight, loss of height, sedentary lifestyle, tobacco use, and hyperthyroidism (Graves’ disease).
Signs and symptoms include severe and localized thoracic-lumbar pain, increased pain with prolonged upright posture, decreased pain in hook-lying, loss of height, and kyphosis (dowager’s hump). Her pain is most likely due to compression fractures, which can be confirmed on x-ray. Osteoporosis can be confirmed with a bone density scan.

180
Q

A patient ambulates with excessive foot pronation. What will the therapist’s examination likely reveal?
A. Varus position of the heel
B. Forefoot valgus
C. Plantar fasciitis
D. Valgus position of the heel

A

Correct Answer: D
Excessive foot pronation is known as pes plans or pes valgus- a “flat foot deformity.” The foot remains in pronation at the subtalar joint during weight bearing. The slight pronation of both the subtalar and tranverse tarsal joints seen in normal stance is exaggerated.

181
Q

A human bite injury resulted in laceration of the extensor tendons over the metacarpophalangeal (MCP) ioints. Following surgical repair, the patient was placed in a dorsal dynamic extension splint (as pictured). Therapy is initiated in the first 24 to 48 hours, with the therapist instructing the patient to move in which way?
A. Actively extend the wrist and passively flex the MCP joints
B. Actively extend the wrist and MCP joints
C. Passively extend the wrist and MCP joints
D. Passively extend the wrist and actively flex the MCP joints

A

Correct Answer: D
Goals during the first few weeks include preventing tendon rupture and promoting tendon healing as well as edema and pain control. For scar management, perform active range of motion (AROM) flexion, isolated joint and tendon gliding (hook and straight fist). Perform passive extension via elastic recoil of the dynamic splint, 10 to 20 reps hourly. Begin active MP flexion to 30 to 40 degrees (via flexion block on dynamic splint). Progress MP flexion as tolerated. Perform wrist and digit passive range of motion (PROM) in extension and tenodesis out of splint 10 repetitions hourly. Avoid making a full fist as this may place too much stress on the repair. The wrist is splinted in 40 to 45 degrees extension with 0-20 degrees of MP flexion and 0 degrees of IP flexion.

182
Q

The patient has venous insufficiency in the right lower extremity resulting in fluid accumulation at the ankle. Which intervention would be most effective at reducing the edema?
A. Crushed ice pack
B. Intermittent compression pump
C. Pulsed ultrasound
D. Contrast bath

A

Correct Answer: B
An intermittent compression pump provides external pressure, increasing the external hydrostatic pressure, which encourages reabsorption of the edema and minimizes fluid outflow from vessels.

183
Q

Following serious trauma, a patient is casted as a result of multiple bilateral wrist and hand fractures. When would it be BEST for physical therapy intervention to begin?
A. As early as possible to maintain or regain strength and range of motion in nonimmobilized joints of the upper extremities
B. As soon as the casts are removed to regain strength of the wrists and hands
C. About 2 weeks after the casts are removed so as not to damage vulnerable soft tissue
D. After 6 weeks of immobilization to ensure that bone healing is almost complete

A

Correct Answer: A
Rehab of the noninvolved joints should begin as early as possible to prevent circulatory stasis. atrophy, and contractures. In addition, exercise to the noninjured area will increase blood flow in the entire extremity and aid in healing and reducing edema. The benefits of range of motion (ROM exercise include prevention of contracture, decreased pain, synovial fluid distribution, cartilage nutrition, stimulation healing along the lines of stress, prevention of thrombus formation, kinesthesia, increased bone activity, and patient participation.

184
Q

What is one of the most common early signs of right ventricular failure?
A. Paroxysmal nocturnal dyspnea
B. Exertional dyspnea
C. Pulmonary edema
D. Dependent edema

A

Correct Answer: D
If the right ventricle fails, the increased fluid will back up. Traveling backward from the right ventricle, the edema goes into the right atrium and then the periphery. This causes dependent edema.

185
Q

A patient with flaccid hemiplegia exhibits pain in the shoulder region secondary to inferior glenohumeral subluxation. Using electrical stimulation as orthotic substitution, where would it be BEST to place the electrodes?
A. Posterior deltoid
B. Supraspinatus
C. Middle deltoid
D. Anterior, middle, and posterior deltoid

A

Correct Answer: B
Electromyography (EMG) studies confirm that the supraspinatus prevents downward migration of the humeral head. Shoulder alignment problems also influence subluxation (a downwardly rotated scapula, relative abduction, and internal rotation of the humerus).

186
Q

An ectomorphic adolescent patient presents at a physical therapy practice with a 6-month history of pain in both hands. Subjective complaints consist of pain that is worse in the morning but gradually improves throughout the day and overall fatigue. The only significant objective data from examination was mild edema and pain at end ranges of motion. What should the physical therapist do next in order to try to establish a diagnosis?
A. Administer the Functional Independence Measure (FIM)
B. Examine for clubbing at the distal interphalangeal joints
C. Examine for Duputren’s contracture
D. Refer the patient to a physician

A

Correct Answer: D
This case represents the typical presentation for juvenile rheumatoid arthritic (JRA)) The following are common symptoms of JRA: swollen, stiff, painful ioints usually worse in the morning; fatigue; fever; swollen Imph nodes; and poor weight gain/slow growth. Additionally, the physical therapist was unable to identify specific impairments that would be potentially contributing to the patient’s complaints, warranting referral to physician for additional testing.

187
Q

A therapist has decided to use mechanical lumbar traction on a patient with posterior herniated nucleus pulposus at L2-3. If tolerated by the patient, what is the BEST positioning for this treatment?
A. Prone, with no pillow under the hips or abdomen
B. Prone, with a pillow under the hips and abdomen
C. Supine, with the hips and knees flexed to 45 degrees
D. Supine, with hips and knees flexed to 90 degrees

A

Correct Answer: A
Neutral or extended position of the spine results in better separation of the disc space and should not exacerbate the posterior herniation.

188
Q

A patient has osteophyte formation affecting the atlantoaxial joint with resultant cervical nerve root irritation. If the physical therapist elects to use gentle, static manual traction to help deal with this problem, what neck position would be best?
A. 10 to 15 degrees of hyperextension
B. Neutral or zero degrees of flexion
C. 20 degrees of flexion
D. 30 degrees of flexion

A

Correct Answer: B
Manual techniques and angle of pull have been determined primarily by palpating where the separation is coming from. However, one should be aware that there is conflicting evidence on angle of pull and vertebral separation. Generally it is believed that C1-2 separation would best be treated with a neutral (0-5 degree)

189
Q

A patient with restrictive lung disease secondary to circumferential thoracic burns demonstrates decreased ability to expand the lower rib cage and push the abdominal wall anteriorly. The therapist should consider the use of facilitation techniques to enhance the function of which of the following?
A. Rectus abdominis
B. Anterior scalenes
C. Internal intercostals
D. Diaphragm

A

Correct Answer: D
Contraction of the diaphragm causes the ribs to move outward, which is desired motion in this case.

190
Q

The picture shows the physical therapist examining the muscle length of the patient’s right hip. How can the muscle length test be interpreted?
A. Decreased length of the rectus femoris
B. Decreased length of the tensor fascia latae
C. Decreased length of the piriformis
D. Normal muscle length of the tensor fascia latae

A

Correct Answer: D
The Ober test is for tightness of the tensor fascia late and iliotibial band. With the pelvis stabilized and the hip kept in neutral rotation, the Ober test would be considered normal if the thigh drops slightly below horizontal. Some sources say that the thigh should drop below the horizontal to the table.

191
Q

A patient with postpolio syndrome (PPS) is referred to physical therapy for exercise training. The patient reports recent general fatigue and weakness along with muscle and joint pain. What is the BEST initial intervention?
A. Treadmill training at 2 mph and a 10 degree slope, 3 days/week for 30 minutes
B. Cycle ergometry at peak heart rate, 3 days/week for 40 minutes
C. Therapeutic aquatics, 3 days/week for 20 minutes
D. Strength training at 70% 1 RM, 2 days/week

A

Correct Answer: C
A good choice for an initial intervention is therapeutic aquatics. The warmth of the water can ease muscle and joint pain, and the buoyancy can assist fatigued limbs. An initial exercise duration of up to 20 minutes per session in 2-to 4-minute intervals is recommended.

192
Q

A patient recovering from stroke walks with limited tibial advancement during stance on the more affected lower extremity. The therapist next examines the patient for a compensatory gait deviation. What is the MOST LIKELY deviation?
A. Trendelenburg
B. Circumduction
C. Exaggerated flexion synergy
D. Exaggerated extension synergy

A

Correct Answer: B
Circumduction is the most likely compensatory gait deviation when tibial advancement is limited (e.g., spasticity of plantar flexors).

193
Q

An elderly patient with a 5-vear history of Parkinson’s disease (PD) demonstrates frequent freezing of gait (FOG) episodes while ambulating. What is the BEST choice of intervention to improve gait and reduce FOG?
A. Part-to-whole training in sequencing of required gait elements
B. Walking using lightly resisted progression with elastic bands to facilitate forward progression
C. Body weight support and treadmill training (BWSTT), 40% unweighting, 3% incline, at
2.7 mph
D. Locomotor training using a personal listening device with 80 to 100 beats/min music

A

Correct Answer: D
Locomotor training using a personal listening device and 80 to 100 beats/min music has been shown to improve rhythmicity and decrease FOG episodes in patients with PD.

194
Q

A patient has limited motion in supination and calcaneal inversion at the subtalar joint.
Using manual techniques. what is the accessory motion of the calcaneus that needs to be emphasized in order to increase the motions that are limited?
A. Anterior glide
B. Posterior glide
C. Medial glide
D. Lateral glide

A

Correct Answer: D
Treatment direction is generally indicated by the convex-concave rule. The convex-concave rule states that when a convex surface moves on a concave surface, the mobilization force is applied opposite to the angular movement of the bone. If the moving surface is concave, the mobilization force occurs in the same direction as bone movement. The non-weight-bearing subtalar joint motion is described strictly by calcaneal movement. Pronation consists of calcaneal eversion, calcaneal dorsiflexion, and calcaneal abduction (medial tilt), whereas supination involves calcaneal inversion, calcaneal plantar flexion, and calcaneal adduction (lateral tilt).

195
Q

A patient has a C5. ASIA A spinal cord injury. An effective coughing technique for this patient is which of the following?
A. Should be facilitated by use of a phrenic nerve stimulator
B. Should be facilitated by use of glossopharyngeal breathing
C. Can be elicited with manual abdominal pressure provided with the assistance of a caregiver
D. Can be elicited with manual abdominal pressure provided independently by the patient

A

Correct Answer: C
The patient will need an external force to facilitate a cough and to clear secretions.

196
Q

A patient with a crush injury to the foot appears to be developing complex regional pain syndrome (CRPS). What are some early signs of this clinical condition the therapist expects to see?
A. Worsening pain with edema and atrophic skin and nail changes
B. Cool, dry, and cyanotic skin with thickened fascia and developing contracture
C. Hyperalgesia, allodynia, and hyperpathia
D. Muscle atrophy, osteoporosis, and developing ankyloses

A

Correct Answer: C
CRPS includes symptoms of pain, vascular changes, and atrophy. Early signs Stage I include hyperalgesia (increased sensitivity to pain), allodynia (all stimuli are perceived as painful), and hyperpathia (increased intensity) with edema, increased sweating, and thin, shiny skin.

197
Q

A patient with long-standing diabetes mellitus is showing early signs of polvneuropathy.
What is the MOST useful test to determine whether demyelination has taken place?
A. Nerve conduction velocity (NCV) testing
B. Electromyography (EMG)
C. Transcutaneous electrical nerve stimulation (TENS)
D. Motor point stimulation

A

Correct Answer: A
NCV provides the most useful measurement of demyelinization in polyneuronathy. Conduction time is measured by recording the evoked potential from either a motor or sensory nerve. Speed of nerve transmission is directly related to level of myelination.

198
Q

During weight bearing, a soft tissue contracture resulting in supination of the forefoot will be compensated for by which of the following?
A. Pronation of the forefoot
B. Pronation of the rearfoot
C. Supination of the rearfoot
D. Pronation of the forefoot and rearfoot

A

Correct Answer: B
When a forefoot is supinated, it can appear to be pronated because the body will get the forefoot onto the ground by pronating somewhere. The pronation that gets the foot on the ground actually occurs higher up the chain as at the rearfoot, ankle, knee, hip or pelvis, and lower back.
When this occurs at the subtalar joint (STJ), for example, it is not uncommon to see the knees achieve a valgus position (knock knees). It is easy to confuse the appearance of pronation in the case of a forefoot varus, where the forefoot is effectively supinated.

199
Q

During a cardiac rehabilitation exercise session involving patients who experienced myocardial infarctions 6 to 8 weeks ago, what is the MOST SIGNIFICANT abnormal response requiring the immediate attention of the physical therapist?
A. Decrease in systolic blood pressure
B. Decrease in diastolic blood pressure
C. Increase in diastolic pressure
D. Increase in heart rate

A

Correct Answer: A
A decrease in systolic blood pressure indicates an inability to maintain cardiac output. Exercise intensity should be immediatelv reduced or exercise stopped, depending upon the patient response.

200
Q

A patient with TB paraplegia sustained 4 ears ago is seen by a physical therapist at a routine outpatient clinic visit. The skin over the ischial tuberosities and sacral region is in perfect shape. The patient asks about the further need for pressure relief in the wheelchair, considering a new gel cushion is being used. What should the therapist advise the patient do?
A. Discontinue pressure relief as long as the gel cushion is used and visual inspection continues to reveal no skin breakdown
B. Continue to perform push-ups in the chair at least once every 15 to 20 minutes
C. Continue to perform push-ups in the chair at least once every 30 to 45 minutes
D. Continue to perform push-ups in the chair once an hour

A

Correct Answer: B
Current recommendations for prevention of pressure ulcers in patients with spinal cord injury who are wheelchair users include 10 to 15 minutes of pressure relief (push-ups) for every 15 to 20 minutes of sitting.